Anda di halaman 1dari 52

SOAL UH B.

INDONESIA KELAS XI 2023


4. Kalimat tanya yang menanyakan alasan
atau sebab adalah ....
A. Mengapa Andi datang terlambat ?
Perhatikan bacaan berikut! B. Apa yang dilakukan oleh Ani setelah
Cara Memperkecil Ukuran PDF, Bisa Tanpa pulang sekolah ?
Aplikasi C. Berapa rupiah uang yang kamu
1. Selanjutnya, klik file kemudian Save As. tabungkan dalam sebulan?
2. Buka PDF di Word menggunakan opsi D. Kapan diselenggarakannya
open dengan di menu file. pertandingan basket antar sekolah ?
3. Pada menu dropdown jenis file, E. Siapa tokoh idolamu?
pilih PDF.
4. Setelah itu cari opsi Optimize for.
5. Klik save dan file PDF sudah terkompres. 5. Ayo berhemat untuk masa depan. Di
6. Pilih opsi minimum size (publishin onlie) daerah Wonogiri, penduduknya sering
mengalami kesulitan air. Hal ini
1. Susunan petunjuk yang disebakan karena memasuki musim
tepat untuk kalimat-kalimat kemarau. Saat musim kemarau, tanah
tersebut adalah.... menjadi tandus, lahan menjadi kering,
A. 2-1-3-4-6-5 dan banyak tumbuhan yang mati.
B. 1-2-3-4-5-6 Pertanyaan yang tidak sesuai dengan isi
C. 2-1-3-4-5-6 teks di atas adalah ....
D. 1-3-2-4-5-6 A. Apa yang terjadi dengan penduduk di
E. 1-2-4-3-5-6 daerah Wonogiri?
B. Bagaimana keadaan saat musim
kemarau tiba?
2. Yang termasuk kalimat persuasif di bawah C. Di daerah manakah terjadi kesulitan
ini adalah. air?
A. Hentikan merokok sebelum candu D. Dimana penduduk di daerah
rokok melekat pada diri anda! Wonogiri mendapatkan air?
B. Ia pergi mengunjungi nenek di E. Musim apa yang sedang berlangsung
kampung. saat itu?
C. Ia bermain HP dari pagi sampai pagi
lagi. 6. Di bawah ini kalimat tanya yang tak perlu
D. Bermain Hp/game bisa tahan bejam- dijawab ialah
jam. A. Apa penyebab kecelakaan itu?
E. Kalau disuruh belajar, langsung B. Apakah Doni kuliah hari ini?
matanya ngantuk. C. Siapa yang akan piket hari ini?
D. Haruskah aku bersujud di hadapanmu?
3. Ciri-ciri kalimat tidak langsung yang E. Yang mana rumahmu?
benar di bawah ini adalah…
A. Tanpa tanda petik, menggunakan 7. Lingkungan hidup adalah segala sesuatu
intonasi datar, sering menggunakan yang ada di sekitar manusia dan
konjungsi bahwa. berhubungan timbal balik. Lingkungan
B. Menggunakan konjungsi tetapi hidup ini mencakupi benda hidup dan
C. Intonasi tinggi di awal, tanpa benda mati. Benda hidup perlu makanan
menggunakan tanda petik, di akhiri dan berkembang biak seperti manusia,
dengan tanda petik dua. binatang, dan tumbuhan. Benda mati antara
D. Diapit oleh tanda petik, dibatasi lain tanah, air, api, batu, dan udara. Jika
dengan tanda koma, menggunakan terpelihara dengan baik, lingkungan hidup
intonasi datar. itu dapat menciptakan masyarakat yang
E. Menggunakan konjungsi bahwa, sehat, aman, tenteram, lahir dan batin.
terjadi perubahan kata ganti, Kata hubung yang terdapat pada teks
dilengkapi intonasi yang bervariasi. laporan hasil observasi tersebut adalah ....

1
11.Manusia pasti memiliki kelenjar keringat
A. Dan yang berjumlah sekitar dua juta. Kelenjar
B. Tetapi tersebut dapat menghasilkan 2.8 liter
C. Sehingga keringat setiap hari. Keringat ini dapat
D. Atau menimbulkan bau yang dapat mengganggu
E. Karena kenyamanan. Bau badan disebabkan oleh
beberapa faktor. Faktor tersebut berasal
8. Indonesia merupakan paru-paru dunia dari diri sendiri maupun dari luar.
kedua. Indonesia memiliki hutan lebat yang Faktor yang berasal dari diri sendiri, yakni
memberikan banyak oksigen. Di negara ini berat badan, keadaan emosional, penyakit
terdapat tumbuhan dan hewan yang khas, fisik, dan kegiatan otot. Berat badan
seperti matoa, kayu cendana, burung seseorang terkadang menentukan jumlah
cendrawasih, orang utan, dan komodo. keringat yang keluar, orang yang memiliki
Kata hubung yang dominan digunakan berat badan lebih akan mengeluarkan
dalam kutipan teks di atas menyatakan banyak keringat. Orang yang baru emosi
hubungan .... atau yang baru melakukan kegiatan otot
A. Penambahan (misalnya, olahraga) juga mengeluarkan
B. Perlawanan banyak keringat. Keringat yang keluar jika
C. sebab-akibat bercampur dengan kuman atau penyakit
D. Pemilihan kulit akan menimbulkan bau yang tidak
E. Syarat sedap.
Faktor dari luar diri sendiri adalah suhu
9. Penggunaan pemakaian titik dua yang tidak udara luar dan kelembaban udara. Udara
benar terdapat pada kalimat... yang cukup panas akan memicu seseorang
A. Yth : H. Syarif, S.S. mengelurkan keringat yang banyak.
B. Hanya ada dua pilihan bagi pelajar :
lulus atau gagal. Teks di atas termasuk jenis teks….
C.Adzkia Kedinasan memiliki dua tentor A. Teks eksplanasi
Bahasa Indonesia kategori mama muda: B. Teks prosedur kompleks
Leni dan Fristi. C. Teks cerita ulang
D. Surah Al-ikhlas :9 D. Teks cerpen
E. Bukan Manusia Biasa: Pak Syarif sangat E. Teks anekdot
Keren

10. Penggunaan tanda titik 12.Bahan; rimpang kencur sebesar ibu jari,
koma yang benar terdapat garam secukupnya cara membuat; kencur
pada kalimat diparut. Tambahkan secangkir air hangat.
A. Beliau lupa akan janjinya ; padahal lalu peras dan disaring. Cara menggunakan
takut apa kata tuannya. kencur yang telah disaring biar mengendap,
B. Ketika dia sudah memimpin dua beri garam secukupnya kemudian diminum.
periode ; malah dia berusaha membuat Kata yang menunjukan urutan kegiatan
kebijakan agar boleh menjabat tiga adalah …
periode. A. Kemudian D. Biar mengendap
C. Bahan yang dibutuhkan yaitu; garam, B. Tambahkan E. Sebesar ibu jari
gula, dan kopi. C. secangkir air
D. Nad-N ad membeli sawi, kol, dan
buncis; baju, celana, dan kaus.
E. Pak Syarif mengajarkan; Bahasa 13. (1) Makhluk hidup terdiri dari
Indonesia; Pak Zul mengajarkan TKP; mikrobiologi, tumbuhan, dan hewan. (2)
Pak Mus mengajarkan TIU. Salah satu ciri makhluk hidup adalah
bergerak. (3) Semua makhluk hidup
memiliki anggota tubuh untuk bergerak. (4)
Pada mikrobiologi dapat memanfaatkan
silia. (5) Tumbuhan pergerakan berupa
pemekaran bunga dan pergerakan akar. (6)

2
Hewan bergerak dengan berbagai alat gerak A. Ramalan pada kalimat pertama
seperti kaki dan tangan, sirip, dan sayap. B. Jawa Tengah pada kalimat kedua
Kalimat aktif transistif terdapat pada C. Selatan pada kalimat kedua
kalimat nomor... D. Bogowonto pada kalimat ketiga
A. 5 D. 2 E. BMKG pada kalimat pertama
B. 6 E. 1
C. 3 16.Bacalah penggalan teks ceramah berikut!
(1) Ada sebuah pepatah yang mengatakan
14.Tari klasik merupakan sebuah tarian yang "Di dalam tubuh yang sehat terdapat jiwa
telah mencapai kristalisasi keindahan yang yang kuat". (2) Pepatah ini menjelaskan
tinggi. Keberadaan yang sudah ada sejak bahwa jika kita memiliki tubuh yang sehat,
zaman dahulu terlahir dan tumbuh di tentunya jiwa dan pikiran kita juga akan
kalangan istana. Salah satu tari klasik menjadi sehat. (3) Ada juga semboyan yang
berasal dari Jawa Tengah yaitu, tari mengatakan bahwa kesehatan sangatlah
Serimpi. Tari Serimpi memiliki gerak mahal harganya. (4) Coba bayangkan
lemah gemulai yang menggambarkan keadaan kita sekarang ini yang masih bisa
kesopanan dan kehalusan budi. Sejak menikmati hidup dan bercengkerama
zaman dahulu, tari ini memiliki kedudukan dengan teman-teman. (5) Lalu,
yang istimewa di keraton-keraton Jawa dan bayangkanlah jika tiba-tiba kita terkena
memiliki sifat yang sakral. Penari tari ini penyakit stroke atau penyakit jantung, pasti
dipilih langsung oleh keraton. Tari klasik sangat tidak mengenakkan, bukan?
menggambarkan tentang bagaimana budaya Penggalan ceramah di atas termasuk ke
Indonesia dapat tetap lestari. Salah satunya dalam bagian....
dengan melestarikan tari Serimpi. Tarian A. penutup D. abstraksi
ini dapat dipentaskan dalam perayaan yang B. pembuka E. simpulan
istimewa. C. isi
Berdasarakan penggalan teks tersebut,
manakah kalimat berikut yang dapat
dipasifkan atau diaktifkan... 17. Perhatikan penggalan teks ceramah
A. Tari klasik merupakan sebuah tarian berikut!
yang telah mencapai kristalisasi Pada kesempatan kali ini saya akan
keindahan yang tinggi. menjelaskan sedikit wacana “sabar”
B. Keberadaan yang sudah ada sejak Sabar berasal dari kata “sobaro-yasbiru”
zaman dahulu terlahir dan tumbuh di yang artinya menahan. Menurut istilah,
kalangan istana. tabah yaitu menahan diri dati kesusahan
C. Tari Serimpi memiliki gerak lemah dan menyikapinya sesuai syariah dan
gemulai yang menggambarkan akal, menjaga mulut dari celaan, dan
kesopanan dan kehalusan budi. menahan anggota tubuh dari perbuatan
D. Tarian ini dapat dipentaskan dalam dosa. Sabar yaitu pilar kebahagian
perayaan yang istimewa. seorang hamba, alasannya yaitu dengan
E. Salah satunya dengan melestarikan tari kesabaran seseorang akan terjaga dari
Serimpi. kemaksiatan, konsisten menjalankan
ketaatan, dan tabah dalam menghadapi
15.Bacalah teks berikut! banyak sekali macam cobaan. Sabar
Berdasarkan ramalan BMKG Pulau Jawa merupakan fatwa yang banyak sekali
akan hujan dengan intensitas ringan sampai disinggung dalam Al-Qur’an maupun
sedang. Di Jawa Tengah bagian selatan hadis, sehingga insan senantiasa
bahkan hujan sudah turun sebelum ramalan diarahkan untuk selalu bersabar dalam
itu disampaikan. Kali Bogowonto yang kehidupannya. Kesabaran yang
melintas di Purworejomeluap hingga bekerjsama yaitu kemampuan dalam
membanjiri permukaan dan persawahan. mengendalikan sikap, sehingga bisa
Banjir tersebut sangat mengganggu dengan nrimo dan rela hati mendapatkan
aktivitas warga. kondisi yang sedang dihadapinya demi
Tanda koma (,) seharusnya di letak setelah menerima jawaban yang baik di akhirat.
kata... Teks tersebut termasuk struktur ….

3
A. Pendahuluan D. Interpretasi Bekasi, Karawang, Subang, dan sebagian
B. Isi E. Orientasi Indramayu, menjadi daerah terbesar di
C. Penutup Indonesia.
Inti kalimat dari paragraf di atas adalah ...
18. Cermati teks berikut! A. Keluasan daerah irigasi Jatiluhur
Sebentar lagi kita akan sampai pada hari B. Daerah irigasi Jatiluhur menjadi daerah
yang bersejarah, yaitu tanggal 10 terbesar
November atau yang disebut dengan Hari C. Terdapat dua jenis saluran, bendungan,
Pahlawan. Pada hari itu kita seluruh dan pintu pembagi di daerah irigasi
bangsa Indonesia akan mengenang Jatiluhur
kembali peristiwa besar sebagai D. Daerah irigasi Jatiluhur mempunyai
momentum sejarah yang terjadi di panjang saluran primer 3.000 km
Surabaya pada tanggal 10 Novenber saluran sekunder, 16 bendungan dan
1945. sekitar 4.000 pintu pembagi
Pronomina persona yang ada dalam E. Daerah irigasi Jatiluhur meliputi
kutipan tersebut adalah... Kabupaten Bekasi, Karawang, Subang,
A. Orang ketiga jamak dan sebagian besar Indramayu
B. Orang kedua jamak
C. Orang kedua tunggal 21. Ayah pergi ke sawah
D. Orang pertama jamak Kalimat di bawah ini yang bukan
E. Orang pertama tunggal transformasi dari kalimat di atas, kecuali....
A. Ayah yang berbaju merah pergi
19. bola Slovania selama ini tidak dianggap B. Ayah Rahmat yang pergi ke sawah
sebagai tim yang perlu ditakuti oleh C. Selain berjualan, ayah yang tangguh
timnas negara Eropa lainnya. Ini wajar tersebut juga bekerja ke sawah.
karena beberapa tahun yang lalu timnas D. Setelah ayah pergi ke sawah, ayah
Slovania ini memang selalu dikalahkan bejualan di pasar.
dengan mudah oleh timnas negara lain di E. Ayah tampan yang pergi ke sawah
benua biru ini. Namun, apa yang terjadi di dengan membawa adik
Kejuaraan Piala Eropa tahun 2016
sungguh di luar perkiraan banyak orang.
Mereka dapat menaklukan timnas negara 22.Perhatikan kalimat-kalimat berikut!
yang sudah punya nama malang (1) Bola terakhir di -- smash oleh Burhan
melintang di Eropa, bahkan cukup sehingga poin nilainya bertambah.
disegani. Bahkan kesebelasan kebanggaan (2) Ayah sudah berangkat menuju Bandara
bangsa Slovania ini maju ke babak Seokarno-Hatta.
semifinal. Timnas Slovania disebut (3) Anak kecil itu berlari-lari mengelilingi lapangan.
sebagai kuda hitam pada Piala Eropa (4) Ilham mengikuti kejuaraan bulu tangkis se-
tahun 2016 itu. Provinsi Jawa Tengah.
Pronomina yang terdapat pada teks di atas (5) Kakakku lahir pada tahun 1990 -- an.
merujuk pada... Kesalahan penggunaan tanda baca terdapat pada
A. Slovania kalimat nomor…
B. Timnas Sepak bola Slovania A. (3) dan (4)
C. Timnas B. (2) dan (4)
D. Kuda hitam C. (1) dan (2)
E. Benua biru D. (1) dan (5)
E. (2) dan (3)

20.Bacalah kalimat berikut! 23.“Kang, kita harus benar-benar pergi dari


Dengan panjang saluran primer 280 km, sini?” Tanya Siti Halimah di sela tangisnya.
3.000 km saluran sekunder, 16 bendungan, “Tentu saja. Seperkasa apa pun perlawanan
dan sekitar 4.000 pintu pembagi, daerah kita, ternyata tetap kalah melawan yang
berkuasa. Kita ini hanya wong cilik, orang
irigasi Jatiluhur, meliputi Kabupaten
iskin,” sahut Karjan sembari melihat rumah

4
Lik Paijan yang siap diruntuhkan. B. Tiara menanyakan di mana sepatu
Teriakan Lik Paijan masuh terdengar baruku.
menyayat hati. Lelaki tua itu merebut tali C. Tiara bertanya di mana sepatu baruku?
yang mengikat seekor sapi miliknya. D. Di mana sepatu baruku? tanya Tiara.
Wajahnya memerah seperti nyaris terbakar, E. Tiara bertanya di mana sepatu barunya.
suaranya melengking-lengking menolak
pengosongan rumahnya. Tetapi, pelawanan
Lik Paijan pun percuma saja. Beberapa
petugas berbadan tegap mengangkat 27.Penggunaan Tanda petik (“...”) yang tidak
tubuhnya. Melihat itu, tangis Siti Halimah tepat terdapat dalam kalimat....
semakin pecah. Dia mendekap Satriya A. Anjani berkata, “Jangan makan di depan
Piningit lebih erat. pintu!”
“Akhirnya kita harus pergi dari rumah kita B. Kak Riza suka memakai celana
sendiri, Kang. Pergi dari kampong yang “cutbrai”.
membesarkan kita,” ucap Siti Halimah C. Bunga membaca majalah “Trubus” hari
getir. ini.
“Iya, mau tak mau kita harus mengalah. D. Bang Toyib memakai teori “coba dan
Gusti Allah tidak tidur, Bune. Di tempat ralat” untuk memakai komputer itu.
lain, semoga kita mendapat ladang rezeki E. “Ayo cepat!” kata Askia, “nanti kita
yang lebih baik lagi,” ujar Karjan. terlambat.”

Sudut pandang dalam kutipan cerpen 28.Penggunaan tanda petik tunggal (‘....’) yang
tersebut adalah …. tidak benar terdapat dalam kalimat, ....
A. Orang pertama tunggal A. Hasifa suka menyanyikan lagu ‘Mardua
B. Orang pertama jamak Holong’.
C. Orang ketiga tunggal B. “Pencuri itu menangis sambil
D. Orang ketiga jamak mengatakan ‘saya terpaksa mencuri obat
E. Campuran ini’, seketika itu amarahku langsung
sirna,” cerita Salwa.
C. “Ustad menjelaskan bahwa ‘hidup ini
24.Manakah penulisan partikel per- yang hanyalah sementara’,” ujar Lesmana.
benar.... D. Cipung berkata, “Semua orang berteriak
A. Harga kain itu Rp2.000,00 perhelai. ‘tolong...tolong’, tetapi semua orang
B. Harga buah itu Rp12.000,00 perkilo. sibuk mengurusi keselamatan diri
C. Mereka masuk ke dalam ruangan satu sendiri.”
persatu. E. Nadia, berkata,” Saya membaca ‘Aku’
D. Per Desember tahun ini, gaji akan naik hari ini.”
E. Perbulan akan diadakan evaluasi.
29.Pada kalimat Hanya surat–surat itulah
25.Penulisan partikel yang tepat adalah ….. yang menghubungkan mereka selama dua
A. Mari lah kita berdoa agar kita tetap cinta tahun terdapat gaya bahasa ….
kepada plajaran bahasa . A. metafora
B. Karena rasa laparnya, makanan apapun B. tautologi
mereka mau C. personifikasi
C. Biar pun jomblo, ia tetap bahagia. D. metonomia
D. Apakah kalian tahu jumlah utang E. anafora
Indonesia yang sebenarnya?
E. Kita akan ujian satu persatu. 30.Setelah mendengar kata-kata baginda, Syah
peri dan Indera bangsawan pun bermohon
26.“Dimana sepatu baruku?”, tanya Tiara untuk mencari buluh perindu itu. Mereka
Jika kalimat tersebut diubah menjadi masuk hutan keluar hutan, naik gunung
kaliamt tidak langsung maka... turun gunung, masuk rimba keluar
A. Tiara menanyakan di mana sepatu rimba.
barunya? Kalimat yang bercetak tebal termasuk
majas…

5
A. Antithesis
B. Antonomasia
C. personifikasi
D. Tautologi
E. Metafora

SEMESTER II

6
IND-1 PROPOSAL C. Pencairan dana bagi penyelenggaraan
KELAS XI kegiatan.
D. Cara-cara untuk melakukan suatu
1. Berikut ini merupakan tujuan dari teks kegiatan.
proposal, kecuali ... E. Cara menyelesaikan sesuatu kegiatan.
A. Untuk mendapatkan bantuan dana.
B. Untuk mendapatkan dukungan. 3. Jenis proposal yang biasa digunakan dalam
C. Untuk mendapatkan perizinan. dunia akademis disebut ...
D. Untuk memperoleh informasi sejelas- A. Proposal bisnis
jelasnya. B. Proposal penelitian
E. Untuk memperoleh bantuan. C. Proposal proyek
D. Proposal kegiatan
2. Pengertian proposal yang tepat adalah ... E. Proposal laporan
A. Teks usulan untuk menyelenggarakan
sebuah kegiatan. 4. Proposal yang terdiri dari tiga bagian, yaitu
B. Laporan hasil untuk terselenggaranya pendahuluan, isi proposal, dan penutup
suatu kegiatan. proposal disebut ...

7
A. Proposal semi formal identitasnya. Tari-tarian itu akan lebih
B. Proposal non formal menarik apabila dipentaskan secara kolosal.
C. Proposal modern Tujuan proposal berdasarkan penggalan
D. Proposal formal latar belakang di atas adalah ...
E. Proposal kuno A. Memasyarakatkan tari daerah
B. Melestarikan tari daerah
5. Teks proposal memiliki kaidah kebahasaan C. Mendemonstrasikan tari daerah
yang membedakannya dengan jenis teks D. Menyelenggarakan pentas tari daerah
lainnya sebagai berikut, kecuali ... E. Mempertontonkan tari daerah.
A. Menggunakan istilah ilmiah, baik yang
berkaitan dengan kegiatan yang akan 9. Untuk mempresentasikan proposal, hal
dilakukan atau yang berkaitan dengan yang tidak perlu diperhatikan adalah ....
bidang keilmuannya. A. Memahami materi yang dipresentasikan.
B. Menggunakan kata-kata yang B. Menyiapkan data yang berkaitan dengan
menyatakan pendefinisian, seperti proposal.
merupakan, yaitu, yakni, adalah. C. Memerhatikan jarak tempat
C. Menggunakan kata-kata denotatif atau dilakukannya presentasi.
bermakna sebenarnya. Hal ini sangat D. Menggunakan bahasa dan sikap yang
penting untuk menghindari santun.
kesalahpahaman antara pihak pengaju E. Menyampaikan materi secara jelas dan
proposal dengan pihak penerima mudah dipahami.
proposal.
D. Menggunakan kata kerja imperatif dan
kata teknis. 10.Ilmu pengetahuan dan teknologi yang
E. Menggunakan kata kerja imperatif dan berkembang pesat saat ini, memunculkan
kata teknis. tuntutan baru dalam segala aspek
6. Pendahuluan dalam proposal berisi ... kehidupan, termasuk sistem pendidikan
A. Penjadwalan kegiatan yang menyangkut pembaharuan sistem
B. Informasi usaha pendidikan. Pembaharuan sistem
C. Kriteria pelanggan pendidikan nasional dilakukan untuk
D. Latar belakang memperbaharui visi, misi, dan strategi
E. Taksasi biaya pembangunan pendidikan nasional.
Bagian proposal tersebut dituangkan
7. Berikut ini merupakan ciri-ciri proposal, dalam ...
kecuali ... A. Masalah
A. Proposal dibuat untuk meringkas B. Pelaksanaan
kegiatan yang akan dilakukan. C. Tujuan
B. Sebagai pemberitahuan pertama suatu D. Latar belakang
kegiatan. E. Penutup
C. Pastinya proposal itu berupa lembaran-
lembaran pemberitahuan yang telah 11. Latar belakang masalah, rumusan
dijilid yang nantinya diserahkan kepada masalah, tujuan penelitian dan hipotesis
si empunya acara. merupakan bagian dari ….. proposal
D. Di bangun atas unsur ADIKSIMBA. penelitian.
E. Memuat unsur-unsur rincian kegiatan A. Bagian
yang akan dibuat. B. ciri-ciri
C. unsur-unsur
8. Kebudayaan merupakan karakter dan jati D. perlengkapan
diri bagi setiap bangsa. Bangsa Indonesia E. pengertian
memiliki keragaman seni budaya, terutama
dalam bentuk tarian yang tersebar di 12. Cermatilah penggalan proposal
seluruh pelosok tanah air, mulai dari berikut ini untuk menjawab soal nomor 8
sabang sampai Merauke, dari Miangas dan 9.
sampai Rote, dengan keunikan dan Teks proposal 1

8
Kasus cyberbullying di media sosial memaparkan efek penyebaran foto
memang tidak bisa dianggap remeh sembarangan.
begitu saja. Aksi mempermalukan orang B. Teks 1 mengungkapkan efek
lain itu bisa berakibat fatal bagi penyalahgunaan narkoba, sementara
korbannya. Dampaknya menyerang teks 2 menerangkan dampak buruk
psikis, mulai dari perasaan malu, merasa manipulasi foto
tertekan hingga depresi. Dengan C. Teks 1 menerangkan perkembangan
maraknya pengguna perangkat mobile psikologi anak, sedangkan teks 2
dan internet, aksi cyberbullying di media memaparkan kecenderungan warganet
sosial dapat dilakukan berulang dan D. Teks 1 mengungkapkan efek
berkelanjutan. Korban akan semakin berlebihan bermain game, sementara
merasa terintimidasi dan ditindas secara teks 2 menerangkan teknik manipulasi
online. Oleh sebab itu, kami selaku OSIS video
SMA Victory bermaksud E. Teks 1 menjelaskan dampak buruk
menyelenggarakan suatu seminar tentang cyberbullying, sedangkan teks 2
dampak cyberbullying di kalangan siswa. memaparkan cara memfoto
Teks proposal 2 sembarangan.
Tidak ada yang salah dengan mem-
posting foto di media sosial. Namun, 14. Cermatilah penggalan proposal
setiap orang harus mengetahui beberapa berikut ini.
fakta ini sebelum mem-posting foto anak Memiliki keterampilan menulis itu tidak
orang lain, atau sebaliknya, membiarkan semudah yang dibayangkan orang.
orang lain men-share foto anaknya. Banyak ahli terampil menuangkan
Pertama, mem-posting foto-foto anak di gagasannya saat berbicara. Namun,
media sosial itu berarti kita sering mereka kurang terampil
meninggalkan rekam jejak digital menyajikannya secara utuh melalui
mereka. Hal itu akan menyebabkan orang tulisan. Oleh sebab itu, perlu kiranya
tersebut malu lantaran fotonya diadakan lomba menulis karya sastra
disebarluaskan sembarangan. Kedua, SMA se-JABAR sebagai wadah bagi
setelah Anda mem-posting foto di media siswa berlatih menulis.
sosial, foto-foto itu tidak lagi dapat kita Penggalan proposal kegiatan di atas
kontrol penggunaannya. Untuk mengatasi merupakan unsur proposal bagian….
persoalan itu, Pegiat Media Sosial A. Pendahuluan
Jakarta ingin menyelenggarakan suatu B. Sasaran
seminar tentang bahayanya menyebar C. Penutup
foto anak di media sosial. D. Jadwal
Persamaan kedua teks proposal tersebut E. Tujuan
ialah….
A. memaparkan simulasi pencegahan 15. Simaklah ilustrasi berikut ini.
manakala terjadi penyebaran berita Dalam rangka memperingati Hari
hoax Sumpah Pemuda, SMA Khoiru Ummah
B. mengungkapkan keburukan yang akan mengadakan kegiatan lomba
ditimbulkan oleh bully di asrama menulis esai dan cerpen antarsekolah.
C. membahas dampak penyalahgunaan Untuk membiayai kegiatan tersebut,
media sosial di masyarakat panitia harus menyusun proposal yang
D. menerangkan teknik manipulasi foto akan disampaikan kepada calon donatur
yang dapat dilakukan oleh setiap untuk menghimpun dana.
orang Kalimat yang menunjukkan tujuan
E. menjelaskan pemakaian media sosial proposal yang sesuai dengan ilustrasi
dengan baik tersebut adalah . . .
A. Dengan lomba menulis esai dan
13. Perbedaan kedua teks proposal cerpen, sikap peduli terhadap
tersebut ialah…. pendidikan bangsa seyogianya
A. Teks 1 menjelaskan dampak buruk ditingkatkan.
cyberbullying, sedangkan teks 2

9
B. lomba menulis esai dan cerpen
menanamkan sikap mencintai
kegiatan menulis, terutama menulis
karya sastra.
C. Lomba menulis esai dan cerpen
merupakan alternatif cara untuk
membuat siswa belajar menulis
karya sastra.
D. Lomba menulis esai dan cerpen
dimaksudkan meningkatkan dan
mengembangkan kreativitas siswa
untuk menulis karya tulis.
E. Lomba menulis esai dan cerpen
merupakan satu-satunya cara untuk
membuat siswa belajar menulis
karya sastra.

IND-2 PENULISAN ANGKA DAN


BILANGAN
KELAS XI

1. Penulisan bilangan yang sesuai dengan


PUEBI terdapat pada kalimat berikut..
A. Liodra dapat mencapai juara ke satu.
B. Pada malam ke-tiga biasanya ada
sesuatu.
C. Lulusan SMA I hari ini akan
berkumpul.
D. Mahasiswa semester ke IX sudah
pantas mendapat peringatan.
E. Indonesia merayakan hari jadi yang
ke-70.

2. Menurut Fajar, orang yang minum


minuman beralkohol kemudian kehilangan
kesadaran maka itu bisa menjadi pemicu
tindak kejahatan. Ia mengutip data Gerakan
Nasional Anti Miras yang menyebut 58
persen kriminalitas di Indonesia itu
disebabkan karena mengonsumsi minuman
keras.
Penulisan bilangan pada kalimat terakhir
dapat diperbaiki menjadi...
A. 58 Persen
B. 58-persen
C. lima puluh lapan persen
D. lima puluh delapan persen
E. Lima puluh delapan persen

10
3. Penulisan bilangan yang benar terdapat C. Dia baru saja mendapat pinjaman
pada kalimat modal usaha lima puluh juta rupiah.
A. 10 orang tewas dalam peristiwa itu. D. Koruptor itu divonis sepuluh tahun
B. Kepala dinas membeli buku itu penjara dan denda sebanyak 4 miliar .
sebanyak 1.000 buah E. Panitia seminar mengundang 350
C. Lempengan baja sebanyak dua ratus peserta dari lembaga pendidikan.
lima puluh lembar telah lenyap.
D. Seng kebutuhan satu toilet sebanyak 7 8. Pada pernyataan tentang penulisan bilangan
kodi. di bawah ini yang paling benar adalah
E. Doli menyaksikan peristiwa itu dua A. 23/5/2022 seharusny 23-5-2022
puluh kali. B. penulisan bilangan 13 dalam kalimat
ditulis tiga belas sudah tepat
4. Penulisan angka/kata bilangan yang tepat C. penulisan 10 seharusnya sepuluh
terdapat dalam kalimat.... D. Rp7.650 seharusnya Rp7.650 (tujuh
A. Tujuh belas orang berhasil dievakuasi ribu enam ratus)
saat peristiwa itu terjadi. E. Rp32.000 seharusnya Rp32.000,00
B. Pandu membeli 10 buah mangga
sebagai oleh-oleh. 9. Penulisan angka dalam kalimat berikut
C. Surat dinas dibuat dalam rangkap 2. yang paling tepat adalah ...
D. Untuk hadiah kepada anak-anak, kami A. Mereka menonton drama itu sampai 3
menyediakan uang 10.000 an. kali.
E. Dua ratus lima puluh orang tamu hadir B. Koleksi perpustakaan itu mencapai 2
dalam acara itu. juta buku.
5. Penulisan angka yang tidak tepat adalah .... C. Kendaraan yang dipesan untuk
A. Bab X, Pasal 5, halaman 252. angkutan umum berjumlah lima puluh
B. Mereka menonton drama itu sampai lima bus, tiga puluh lima mini bus.
tiga kali. D. Lima puluh siswa kelas enam lulus
C. 50 siswa teladan mendapat beasiswa ujian.
dari pemerintah daerah. E. 250 orang peserta di undang ke acara
D. Perusahaan itu baru saja mendapat seminar.
pinjaman 550 miliar rupiah.
E. Di bantaran Sungai Ciliwung 10. Kalimat berikut yang tidak menggunakan
sedikitnya ada 71.000 keluarga atau penulisan kata bilangan sesuai EYD
350.000 jiwa yang hidup di daerah itu. adalah . . .
A. Ibu membeli 20 kg beras di pasar.
6. Di antara 72 anggota yang hadir, 52 orang B. Penghuni rumah itu ada sepuluh
setuju, 15 orang tidak setuju, dan lima orang.
orang memberikan suara blangko. C. Di kelas kami hanya terdapat enam
Pernyataan yang benar sesuai EBI pada kelompok.
kalimat tersebut terdapat pada . . . D. Adik memiliki 2 buah pulpen, 13
A. Di antara seharusnya Diantara. pensil, dan 25 buah buku tulis.
B. Lima seharusnya 5. E. Di kelas kami hanya terdapat 6 cewek
C. Angka 72 dan 52 harusnya tujuh puluh cantik.
dua dan lima puluh dua.
D. Kata blangko seharusnya belangko. 11. Penulisan ejaan yang tepat dengan
E. Semua salah menggunakan angka adalah . . . .
A. Harga kopi itu Rp6000,00/kg.
7. Berikut ini penulisan bilangan yang sesuai B. Tahun ini kita memperingati HUT ke 63
dengan PUEBI adalah .... Kemerdekaan RI.
A. 2000 orang massa akan melakukan C. Di rumah kami terdapat 9 buah kamar.
demo di depan kantor Gubernur Jawa D. Di dalam tasnya ditemukan tiga potong
Barat. celana, dua bungkus permen, dan 25
B. Dua puluh lima orang mahasiswa akan bungkus rokok.
berangkat ke Bangkok.

11
E. Adik memiliki 2 buah mobil-mobilan, 2 15.Di antara 95 peserta yang mendaftar, 32
tembak-tembakan, dan 20 buah kapal- orang lulus, 55 tidak lulus , dan delapan
kapalan. orang tidak hadir.
Kesalahan pada kalimat tersebut, kecuali....
12.Penulisan bilangan yang tidak sesuai A. Angka 95 harusnya sembilan puluh
dengan yang digariskan dalam EYD lima
dijumpai dalam B. Angka 32 harusnya ditulis tiga puluh
kalimat … dua.
A. Dari hasil penelitian yang dilakukan C. Delapan seharusnya 8.
Yayasan Kusuma Buana di sekolah- D. Di antara seharusnya Diantara
sekolah di Jakarta, Prevalensi anemia di E. 55 seharusnya lima puluh lima
setiap sekolah antara 20 persen dan 35
persen, bahkan ada yang mencapai 60
persen.
B. Sekolah Menengah Kejuruan Negeri
(SMKN) X, dua bulan yang lalu,
dikunjungi oleh 20guru dan 5 pejabat
Pemerintah Malaysia.
C. Penghargaan Festival Teater
diperebutkan oleh 18 finalis yang
tampil di Teater Luwes InstitutKesenian
Jakarta dan Teater Studio Taman Ismail
Marzuki.
D. Dari 422 laporan yang masuk ke Komisi
Kejaksaan, sebanyak 202 laporan
dilanjutkan keJaksa Agung.
E. Dari tiga ratus laporan masyarakat
yang masuk ke Komisi Kejaksaan
pada kurun April –Desember 2006, IND-3 HURUF KAPITAL
yang diteruskan ke Jaksa Agung KELAS XI
sebanyak dua ratus laporan.
1. Bacalah teks berikut untuk menjawab
13.Penulisan lambang bilangan dengan benar soal nomor 1.
terdapat dalam kalimat . . . Di indonesia dan di semua negara
A. Peristiwa itu terjadi sekitar tahun 50 an. dengan sistem politik demokrasi, membagi-
B. Tolong kamu pelajari halaman tiga bagi jabatan kepada parpol pendukung dan
puluh lima. tim sukses adalah kelaziman. Sejak masa
C. Angka 222 yang benar ditulis dalam orde lama hingga orde reformasi, bancakan
huruf menjadi dua ratus dua puluh dua. jabatan dan kekuasaan sudah dan terus
D. Paku Buwono X, Paku Buwono ke – 10 dilakukan.
atau Paku Buwono kesepuluh. Namun, di masa rezim sekarang
E. Dia menginap di Hotel Indonesia, praktik bancakan ini dilakukan begitu
Kamar seratus enam puluh sembilan. vulgar, tanpa malu. Pertama, Presiden
sempat berjanji kabinetnya tidak akan diisi
14.Penulisan angka yang TIDAK benar dengan praktik bagi-bagi jabatan. Akhirnya
terdapat pada kalimat . . . ia membagi-bagi jabatan seperti jabatan
A. Perjalanan ke daerah terpencil itu menteri dan komisaris BUMN pada Partai
membutuhkan 3.32.30. Politik pendukung dan tim suksesnya.
B. Dia beralamat di Jalan Tanah Abang I Kedua, Pemerintah juga membagi jabatan
No. 15. untuk puluhan perwira kepolisian. Kontras
C. UUD’45, Bab 10, Pasal 5, Ayat 1. mendata ada 30 perwira yang mendapatkan
D. Saya bertemu dengannya dua kali. jabatan di era saat ini mulai dari komisaris
E. Dia mempunyai 10 angkot dan 50 hingga pejabat, di mana 21 perwira masih
bemo. aktif di kepolisian.

12
Kesalahan pemakaian huruf kapital terdapat C. antara pemerintah dan rakyat
pada bacaan di atas, kecuali.... D. pengesahan rancangan undang-undang
A. kalimat pertama paragraf pertama kepegawaian
B. kalimat kedua paragraf pertama E. berdasarkan undang-undang yang
C. kalimat kedua paragraf kedua berlaku
D. kalimat ketiga paragraf kedua
E. kalimat keempat paragraf kedua 6. Penulisan judul yang sesuai dengan aturan
EBI yang baku adalah...
2. Kata bercetak miring yang tidak perlu A. Rahasia Guru Berkualitas Dan Cantik
ditulis dengan huruf awal kapital terdapat Bawaan Lahir Sebagai Teladan
dalam kalimat.... B. Rahasia Guru Berkualitas dan cantik
A. Truk besar samapai dikerahkan untuk Bawaan Lahir sebagai Teladan
mengeruk tumpukan sampah di Daerah C. Rahasia Guru Berkualitas dan Cantik
Perairan Indonesia. Bawaan Lahir Sebagai Teladan
B. Tambang minyak di Provinsi Papua, D. Rahasia Guru Berkualitas Dan Cantik
menjadi sorotan dunia. Bawaan Lahir sebagai teladan
C. Suara warga Kampung Batak menyita E. Rahasia Guru Berkualitas dan Cantik
perhatian musisi mancanegara. Bawaan Lahir sebagai Teladan
D. Truk pengangkut barang dermaga di
Pelabuhan Bakauheni rusak. 7. Penulisan judul yang benar adalah....
E. Pemerintah Sumatra Utara menguji A. Pudarnya Minat Remaja terhadap Tarian
coba layanan perpanjangan KTP melalui Tradisional
android. B. Pengaruh Olahraga bagi generasi muda
C. Peranan Negara Dalam Membina
3. Penulisan sapaan yang benar menurut ejaan Kerukunan Beragama
yang baku terlihat pada kalimat.... D. Usaha-usaha Pembinaan Karakter Di
A. “Tunggu sebentar, silakan duduk dulu, Indonesia
pak!” E. Manfaat Kesenian Daerah Bagi
B. “O, jadi Anisa itu sepupu saudara?” Masyarakat Toraja
C. “Apakah keterangan Ibu dapat saya
jadikan pegangan?” 8. Penggunaan huruf kapital yang benar
D. “Luar biasa, Anak ibu memang cerdas.” terdapat dalam kalimat....
E. “Jika kakak belum percaya, kakak dapat A. Di mana banyak Suku Tionghoa?
melihat sendiri daftar nama di sebalik B. Pada Bulan Ramadan ia akan berangkat
kertas ini!” ke Arab Saudi.
C. Dengan gembira, seluruh masyarakat
4. Penulisan huruf kapital yang benar menyambut lebaran.
berdasarkan PUEBI terdapat dalam D. Kita harus berusaha menggunakan
kalimat... bahasa Indonesia yang baik dan benar.
A. Brigadir Jenderal Haqqie akan dilantik E. Setiap Hari Sabtu, bus Jakarta menuju
menjadi mayor Jenderal. Semarang penuh sesak.
B. Brigadir Jenderal Haqqie akan dilantik
menjadi mayor jenderal. 9. Kalimat yang mengandung kesalahan
C. Brigadir jenderal Haqqie akan dilantik dalam penulisan huruf adalah....
menjadi mayor jenderal. A. Siapakah gubernur yang baru dilantik?
D. Brigadir jenderal Haqqie akan dilantik B. Kemarin Brigadir Jendral Sayid dilantik
menjadi Mayor Jenderal. menjadi mayor.
E. Brigadir jenderal Haqqie akan dilantik C. Kehidupannya dari kecil sudah seperti
menjadi mayor Jenderal. anak sultan.
D. Gubernur Irian Jaya baru saja
5. Huruf kapital seharusnya dipakai pada kata menghadap Presiden.
atau frasa yang bercetak miring dalam E. Surat Anda telah saya terima..
pilihan...
A. dalam sebuah republik 10.Penulisan huruf kapital yang tidak sesuai
B. membicarakan beberapa badan hukum dengan EBI terdapat dalam kalimat

13
A. “Silakan duduk, Bu!” kata Audi kepada Muhammad itu nama Nabi dan sosok yang
orang itu. sangat dihormati oleh Umat Islam. “Beliau
B. Ayahnya bertanya, “Adikmu sudah telah melarang umatnya untuk meminum
pulang?” alkohol dan atau minuman keras, tapi
C. Abangnya pernah menjadi Wakil Lurah Holywings malah memberi hadiah dan
terbaik. menggratiskan satu botol alkohol bagi
D. Abangnya yang paling tua sekarang calon konsumen yang memiliki nama
sudah bekerja di Jakarta. muhammad,” kesalnya.
E. Departemen itu memiliki beberapa Mungkin, lanjut Anwar, ada yang
direktorat jenderal. berkilah bahwa mereka tidak menghina
Nabi Muhammad dan orang yang beragama
11. Penulisan yang tidak tepat terdapat pada Islam. Mereka hanya ingin memberi
kalimat, kecuali kehormatan dan hadiah kepada orang yang
A. Danau Toba mengelilingi pulau bernama Muhammad. “Pertanyaannya dari
Samosir. ribuan nama yang ada yang mungkin bisa
B. Kami sedang menganalisis peranan mereka pilih dan kasih hadiah mengapa
Bahasa Jawa bagi Suku Jawa. nama Muhammad yang mereka pilih?
C. Setiap Hari Jum‘at mereka selalu jalan- Apakah mereka tidak tahu nama itu adalah
jalan. nama yang sangat dihormati oleh umat
D. Profesor itu bernama Nisrina N Nabila. Islam?” tanyanya. “Saya melihat hal ini
E. Pak Syarif adalah guru berdarah Batak. jelas-jelas sangat tendensius dan berpotensi
memancing kekeruhan dan kemarahan
12. Bacalah teks berikut! umat Islam. Tapi karena yang bersangkutan
Menyikapi banyaknya perbuatan yang sudah minta maaf ya sudah,” tandasnya. Ia
menyerempet SARA yang cukup menilai, tindakan-tindakan tersebut di atas
mengganggu dan menimbulkan kegaduhan. tentu menyisakan pertanyaan. “Mengapa
Wakil Ketua Majelis Ulama Indonesia, akhir- akhir ini kok sering sekali muncul
Buya Dr. Anwar Abbas mempertanyakan tindakan dan perilaku yang
sikap aparat keamanan. “Kenapa pihak mendiskreditkan agama Islam dan
keamanan kesannya kok biasa-biasa saja? umatnya?” tanyanya.
Yang saya tahu kesabaran seseorang atau Dari bacaan di atas, manakah kalimat yang
sekelompok orang itu ada batasnya. Dan tidak terdapat kesalahan penulisan huruf
bila batas itu dilewati maka tidak mustahil kapital?
hal-hal yang tidak kita inginkan bisa terjadi A. kalimat pertama paragraf ketiga
dan hal demikian tentu saja tidak kita B. kalimat pertama paragraf kedua
harapkan,” tuturnya pada Mediaumat.id, C. kalimat kedua paragraf ketiga
Sabtu (25/6/2022). D. kalimat kedua paragraf kesatu
Anwar mencontohkan beberapa E. kalimat ketiga paragraf keempat
perbuatan SARA tersebut, seperti adanya 13. Bacalah teks berikut!
orang atau pihak yang membuat rendang Kedutaan Besar Inggris baru-baru ini
daging babi Padang dan nasi uduk babi mengibarkan bendera pelangi khas LGBT
Aceh yang sangat menyinggung dan di Indonesia. Menurut mereka, pengibaran
melukai hati orang Minangkabau atau bendera tersebut untuk memperingati Hari
Padang dan Aceh.“Dan terakhir kita Internasional Melawan Homofobia,
dikejutkan lagi oleh Holywings di akun Transfobia, dan Bifobia. Kedubes Inggris
Instagramnya yang jelas-jelas mengandung juga mengatakan bahwa setiap orang
unsur SARA,” ujar Anwar merujuk kafe memiliki hak untuk mencintai siapa pun
yang sempat menampilkan gambar dari dan bebas untuk mengekspresikan diri
promosi produk minuman beralkoholnya mereka.
dengan menggratiskan satu botol alkohol Sikap Kedubes Inggris ini sudah
bagi calon konsumen yang memiliki nama sepantasnya dikecam. Wakil ketua MPR
Muhammad dan Maria. menyebut tindakan Kedubes Inggris itu
Menurut Anwar, hal ini tentu jelas provokatif, bisa memantik masalah. Bahkan
sangat menyinggung hati Umat Islam. Ia tindakan itu bisa dinilai sebagai penjajahan
memberikan alasan karena nama HAM.

14
Kesalahan pemakaian huruf kapital pada
bacaan di atas terdapat pada...
A. kalimat pertama paragraf pertama
B. kalimat kedua paragraf pertama
C. kalimat kedua paragraf kedua
D. kalimat ketiga paragraf kedua
E. kalimat keempat paragraf kedua

14. Penulisan sapaan yang benar menurut ejaan


yang baku terlihat pada kalimat....
A. Atas perhatian Bapak, saya
mengucapkan terima kasih.
B. Oh, jadi Nazla Oscalino itu sepupu
anda?
C. Luar biasa, Anak saudara memang perlu
perhatian lebih.
D. Jika Ibunya datang, silakan berdiri
terlebih dahulu!
E. Besok pagi Kakaknya datang.

15. Kendati tetap mempertahankan lirik-lirik


yang puitis, bermakna, dan kontemporer,
album yang digarap mulai Februari itu
memang berbeda dengan album Bocil
sebelumnya yang penuh dinamika. Pada
album kali ini, musik tanah air berhasil
memberikan kesan yang sangat menonjol
pada konser terakhir menyambut Bulan
Ramadan. Lagu "Seandainya Bisa Menjadi
Orang Biasa" dan "Syukurku Menjadi
Hamba" misalnya, sangat menunjukkan
kemampuan vokal seorang penyanyi
sekaligus musikus papan atas itu dalam IND-4 PENULISAN GABUNGAN KATA
menyesuaikan tempo musik yang berubah- DAN PREPOSISI
ubah. Nuansa harpa dari seorang harpais KELAS XI
kebanggan Indonesia serta paduan suara
turut memperkaya kesan orkestra musik 1. Dengan UU Cipta Kerja, pemerintah
yang mengiringi konser sukses yang seolah kontra produktip dengan filosofi
glamor dari seorang multitalenta, Bocil, dari upah minimum, yakni sebagai jaring
untuk kesekian kalinya. pengawas agar masyarakat tidak miskin
Kesalahan penulisan huruf kapital terdapat secara absolut.
pada kalimat ... Penulisan kata kontra produktip pada teks
A. satu di atas masih kurang tepat, perbaikan yang
B. dua paling tepat adalah...
C. tiga A. kontra produktif
D. empat B. kontraproduktip
E. lima C. kontraproduktif
D. kontra-produktif
E. kontrakan produktif

2. Penulisan kata yang ditulis dengan tepat


terdapat dalam kalimat...
A. Proyek itu diberikan kepada sub
kontraktor lain.

15
B. Sinar infra merah saat ini sudah banyak E. bayi tabung, terima kasih, buku harian
diketahui.
C. Makanan nonkolestrol sangat dianjurkan 6. Penulisan kata yang sesuai dengan ejaan
untuk lebih diprioritaskan. yang baku terdapat pada kalimat....
D. Kondisi jiwa remaja pasca SMA sudah A. Kamu harus mempertanggungjawabkan
stabil. perbuatanmu.
E. Keputusan saat ini kontra produksi B. Dia memberitahu masalah itu
dengan hasil pernyataan sebelumnya. kepadanya.
C. Mereka tidak diberi tahukan atas
3. Penulisan gabungan kata yang salah peristiwa itu.
terdapat pada kalimat D. Tolong beritahu saya!
A. Ayah membeli saputangan merah. E. Pemberi-tahuan itu datang terlambat.
B. Pesta olahraga itu akan diadakan di
Papua. 7. Penulisan gabungan kata yang tepat
C. Kita harus tanggungjawab pada semua terdapat dalam kalimat....
yang sudah kita putuskan. A. Angkutan antar kota banyak yang tidak
D. Hari ini akan diadakan serah terima layak pakai.
jabatan. B. Masalah poli gami sedang naik daun.
E. Aktivitas mahasiswa saat ini sangat C. Kegiatan ekstrakurikuler lebih banyak
baik. diminati daripada olimpiade.
D. Penjualan minyak non dubsisi dilakukan
4. Ketua Bidang Komunikasi dan Media KSPI bulan Juli.
Pusat menilai Omnibus Law tidak hanya E. Hal itu merupakan tanggungjawab kita
bicara tentang tenaga kerja, tetapi bicara bersama
tentang masyarakat Indonesia. “Di situ ada
subklaster ketenagalistrikan. Sub pasal 8. Penulisan bentuk kata gabung yang tepat
dalam ketena galistrikan itu menghidupkan terdapat pada kalimat…
kembali pasal yang pernah diuji dalam A. Wistawan manca negara akan
Mahkamah Konstitusi pada saat itu,” berkunjung ke Indonesia bila keadaan
ujarnya. Menurutnya, Mahkamah aman
Konstitusi pernah menegaskan bahwa B. Hubungan antar negara sangat terbuka
listrik itu hajat hidup orang banyak. ―Oleh pada era globalisasi
sebab itu, listrik harus dikuasai oleh negara C. Perdagangan agro bisnis tidak
supaya harga listrik tidak dikendalikan terpengaruh oleh krisis ekonomi
swasta,‖ ungkapnya. Dengan demikian, D. Pengembangan pasca sarjana harus
menurutnya masyarakat akan mendapatkan digalakkan untuk mendapatkan nilai
harga listrik yang murah karena di tambah
bawah kendali negara. “Tapi ternyata, kita E. Subsidi silang sangat diperlukan untuk
melihat didalam Omnibus Law ini akan menyantuni penyandang tunanetra
diserahkan publik,” pungkasnya.
Pada paragraf di atas, penulisan gabungan 9. Penggunaan preposisi yang benar terdapat
kata yang salah adalah.... dalam kalimat di bawah ini, kecuali..
A. ketenaga listrikan A. Mereka langsung mengajukan banding
B. subpasal atas putusan hakim tersebut
C. subklaster B. Bus itu hampir sampai ke tujuan.
D. menghidupkan C. Belajar itu lebih baik dari main game.
E. mendapatkan D. Ia belajar dengan tekun.
E. Penjualan tanah itu disaksikan oleh
5. Kata-kata di bawah ini adalah gabungan kepala desa.
kata yang benar penulisannya, kecuali
A. simpang empat, sumber daya, mata 10. Penggunaan preposisi yang benar terdapat
pelajaran dalam kalimat dibawah ini, kecuali....
B. matahari, halalbihalal, bumiputra A. Ia pun suka mengatur meja makan
C. halal bi halal, sapu tangan, kaca mata untuk rekan-rekannya.
D. ekstrakurikuler, dasawarsa, acapkali

16
B. Aliran air terus menerus masuk, antara E. (1)
lain melalui lubang di bawah pintu dan
lubang yang lain. 12.Penulisan gabungan kata yang tidak tepat
C. Lebih dari lima puluh ladang ranjau terdapat pada kalimat berikut, kecuali....
ditemukan di sekitar Kabul. A. Garisbawahilah kata-kata yang tidak
D. Ibu atau bapak itu harus ke sana- kau pahami
kemari di dalam kota untuk mencari B. Pada hari libur bali dipenuhi wisatawan
makan sekadarnya manca Negara
E. Kesadaran masyarakat semakin besar C. Dari situlah muncul ketidaksehatan
kepada masalah ini. alam pikiran manusia
D. Anakanya menjadi pengemudi bis antar
11. Bacalah teks berikut dengan cermat. kota
(1) Saat virus corona mulai mewabah di E. Ia menjadi juara dasa lomba di
Wuhan, Cina, akhir tahun 2019, banyak sekolahnya
penelitian yang menyebutkan bahwa
distribusi satwa liar dalam jalur 13.1.sapu tangan 2.beasiswa
perdagangan ilegal menjadi salah satu 3.matahari 4.segi tiga
penyebab kemunculannya. (2) Namun, 5.belasung kawa 6.olahraga
kemunculan wabah adalah proses yang 7. sukacita 8.darmabakti
kompleks. (3) Sebagai peneliti dalam Kata gabung di bawah ini yang benar
bidang biologi konservasi, kami penulisannya adalah...
menganalisis hubungan munculnya A. 1,2.3
emerging diseases, atau penyakit baru B. 1, 2, 4
pegari, seperti COVID-19, bagi manusia C. 2, 3, 4
dengan pengelolaan satwa dan habitat yang D. 3, 6, 7
tidak benar. (4) Penyakit baru pegari E. 2, 5, 8
hampir selalu bermula dari perambahan
habitat dan praktik pertanian yang tidak 14.Penggunaan dan penulisan preposisi yang
ramah lingkungan yang dilakukan oleh benar terdapat dalam kalimat di bawah ini,
manusia. (5) Penelitian terbaru dari kecuali..
Stanford University, Amerika Serikat A. Ia mencari makan untuk menghidupi
menemukan bahwa berkurangnya luas keluarganya.
hutan di Uganda berisiko meningkatkan B. Anita merasa suka pada Syandri sejak
interaksi manusia dengan primata liar, duduk di bangku SMP.
meningkatkan risiko kontak manusia C. Tugas itu dikerjakan dengan baik oleh
dengan virus yang ada pada primata liar Budi.
tersebut. (6) Beberapa wabah penyakit D. Gelas itu lebih baik dari pada gelas
zoonosis, penyakit yang ditularkan melalui yang kemarin dibeli.
satwa liar, tercatat terjadi di Uganda, E. Pekerjaan tersebut cepat selesai berkat
seperti virus Ebola dan virus Marburg. (7) bantuan yang diberikan Pak Syarif.
Kedua virus ini dapat menginfeksi baik
manusia maupun kera, menyebabkan 15.Penggunaan dan penulisan preposisi yang
pengidapnya mengalami demam yang benar terdapat dalam kalimat di bawah ini,
diikuti oleh pendarahan dalam. (8) kecuali..
Singkatnya, semakin banyak manusia A. Masalah berat yang dialami anita
membabat hutan untuk bertani atau sampai membuatnya stres.
membangun infra struktur, semakin tinggi B. Ia mempelajari tentang bahasa
risiko manusia berinteraksi dengan hewan Indonesia.
yang membawa virus. C. Keputusan itu diambil untuk
Kesalahan penggunaan gabungan kata pada meningkatkan kesejahteraan bersama.
teks di atas ditunjukan pada nomor... D. Demi kebahagiaan anaknya, ibu itu
A. (8) ikhlas anaknya merantau jauh.
B. (5) E. Pewarna itu ditambahkan hingga
C. (3) adonan berubah warna.
D. (2)

17
IND-5 KARYA ILMIAH
KELAS XI

1. Ragam bahasa yang digunakan karya


ilmiah haruslah....
A. lugas D. formal
B. tepat E. ganda
C. ringkas

2. Di bawah ini yang termasuk unsur-unsur


karya ilmiah adalah ...
A. judul, lampiran, kata pengantar, daftar
isi, kesimpulan, daftar pusaka,
glosarium.
B. judul, daftar isi, halaman persembahan,
halaman pengesahan, kata pengantar,
abstarak, pendahuluan, kerangka teori,

18
metode, pembahasan, kesimpulan, membebaskan mereka dari bencana
daftar pustaka. Gunung Merapi dan gempa bumi serta
C. judul, biografi penulis, kata pengantar, tsunami di Indonesia.
daftar isi, kesimpulan, glosarium. C. Masyarakat Yogyakarta ingin
D. judul, daftar isi, halaman persembahan, memulihkan kepercayaan pemerintah
halaman pengesahan, koda, abstarak, kepada mereka dengan mengadakan
pendahuluan, kerangka teori, metode, kirab budaya sehingga wisatawan yang
pembahasan, kesimpulan, daftar akan datang kembali.
pustaka. D. Aktivitas Gunung Merapi dan terjadinya
E. judul, halaman judul, kata pengantar, gempa bumi akhir-akhir ini telah
daftar judul, daftar isi, halaman mengganggu Masyarakat yogyakarta
persembahan, halaman pengesahan, kata sehingga pemerintah perlu mengadakan
pengantar, abstarak, pendahuluan, kirab budaya untuk memohon
susunan acara, metode, pembahasan, keselamatan.
kesimpulan, daftar pustaka. E. pemerintah memandang perlunya
mengadakan kirab budaya di Indonesia
3. Penulis menyadari bahwa karya tulis ini untuk memulihkan mental masyarakat
masih jauh dari sempurna. Oleh karena itu, yang terkena bencana alam di Indonesia.
penulis mengharapkan kritik dan saran
yang membangun dari semua pihak. 6. Sebuah karya ilmiah diolah berdasarkan
Bagian karya tulis di atas terdapat pada berbagai ketentuan yang logis. Salah satu
bagian ... pondasi karya ilmiah perlu ada latar
A. Saran belakang masalah. Latar belakang masalah
B. Isi tersebut terdapat pada bagian ...
C. pendahuluan A. inti masalah
D. kata pengantar B. pemecahan masalah
E. simpulan C. kesimpulan dan saran
D. pendahuluan
4. Judul karya tulis: menggali potensi diri E. perumusan masalah
untuk meningkatkan kepercayaan diri.
Penulisan judul makalah yang tepat 7. Kualitas anak-anak, pemilik masa depan,
adalah ... sangat ditentukan oleh perlakuan kita
A. Menggali Potensi Diri Untuk terhadap mereka saat ini. Maju mundurnya
Meningkatkan Kepercayaan Diri suatu bangsa di masa depan sangat
B. Menggali potensi diri untuk dipengaruhi oleh kesiapan fisik, mental,
meningkatkan kepercayaan diri sosial, maupun intelektual generasi yang
C. Menggali Potensi Diri untuk saat ini masih berada dalam fase kanak-
Meningkatkan kepercayan diri kanak. Dalam karya tulis, uraian di atas
D. Menggali Potensi Diri untuk merupakan bagian ...
Meningkatkan Kepercayan Diri A. tujuan penulisan
E. Menggali Potensi Diri untuk B. landasan teori
Meningkatkan Kepercayan Diri. C. latar belakang
D. permasalahan
5. Tema karya tulis : Kirab Budaya sebagai E. pembatasan masalah
Terapi Mental untuk Memulihkan
Kepercayaan Masyarakat Yogyakarta. 8. Unsur-unsur yang harus ada dalam catatan
Latar belakang yang tepat untuk tema kaki, kecuali ...
tersebut adalah.... A. nomor catatan kaki
A. Kirab budaya adalah pawai dengan B. tahun terbit
memberi persembahan ke Gunung C. nama pengarang
Merapi dan Laut selatan agar terhindar D. judul buku
dari bencana yang lebih besar dan lebih E. kata pengantar buku
mengerikan.
B. Kirab budaya merupakan acara yang
dipercaya masyarakat dapat

19
9. Berikut adalah identitas pustaka yang E. Rahmini P. 2008. Pendidikan Anak
dapat digunakan sebagai sumber untuk Jalanan. Surabaya : Bintang.
menulis karya ilmiah Puspitasari, P. 2007. Memanusiakan
Judul Penulis Kota Penerbit Tahun Anak Manusia, Medan : Cahaya.
Pendidika Sugentar, Ahmad. 2008. Perdagangan
Rahmini. Anak. Ambon : Cemerlang.
n Anak SurabayaBintang 2008
P Rohe, Siana. 2009. Bimbing Anak
Jalanan
Memanu Berakhlak Mulia. Surabaya : Gemerlap.
P.
siakan
Puspitasar Medan Cahaya 2007 10.Pembelajaran berbasis teknologi komputer
Anak
i perlu segera disosialisasikan sampai ke
Manusia
Perdaga sekolah – sekolah di pelosok tanah air.
Ahmad Cemerlan Pernyataan tersebut terdapat dalam buku
ngan Ambon 2008
Sugentar g “Pembelajaran Berbasis TIK” halaman 32
Anak
Bimbing karya Rahmat Arif yang diterbitkan oleh
Anak Dr. Siana Surabay Penerbit Fantasi pada tahun 2009. Teknik
Gemerlap 2009 pengutipan teks di atas yang betul adalah ...
Berakha Rone a
k Mulia A. “Pembelajaran berbasis teknologi
komputer perlu segera disosialisasikan
Azhari menulis daftar pustaka untuk
ke sekolah – sekolah di pelosok tanah
karangannya yang berjudul Pembinaan
air” (dalam Arif, 2009: 32).
Anak Jalanan. Berdasarkan data pustaka
B. Menurut Arif (2009: 32) adalah
di atas, penulisan daftar pustaka yang
“Pembelajaran berbasis teknologi
relevan dan tepat untuk karangan Azhari
komputer perlu segera disosialisasikan
tersebut adalah . . .
sampai dengan sekolah – sekolah
A. Ahmad Sugentar. 2008. Perdagangan
pelosok tanah air.”
Anak. Ambon : Cemerlang.
C. Menurut Arif (2009: 32) bahwa
P. Puspitasari. 2007. Memanusiakan
pembelajaran berbasis teknologi
Anak Manusia. Medan : Cahaya.
komputer perlu segera disosialisasikan
Rahmini P. 2008. Pendidikan Anak
ke sekolah – sekolah di pelosok tanah
Jalanan. Surabaya : Bintang.
air.
Siana Rohe. 2009. Bimbing Anak
D. “Pembelajaran berbasis teknologi
Berakhlah Mulia. Surabaya :
komputer perlu segera disosialisasikan
Gemerlap.
sampai dengan sekolah – sekolah
B. Puspita, P. 2007. Memanusiakan Anak
pelosok tanah air” (Arif, 2009: 32)
Manusia. Medan: Cahaya.
E. Arif (2009: 32) “Pembelajaran berbasis
Rahmini, P. 2008. Pendidikan Anak
teknologi komputer perlu segera
Jalanan. Surabaya : Bintang.
disosialisasikan ke sekolah – sekolah di
Rohe, S. 2009. Bimbing Anak
pelosok tanah air.”
Berakhlak Mulia. Surabaya : Gemerlap.
Sugentar, Ahmad. 2008, Perdagangan
11. Dalam buku Jangan Serahkan Anak
Anak, Ambon : Cemerlang.
kepada Pembantu halaman 11 karya
C. Puspitasari, P. 2007. Memanusiakan
Imron Zamani yang terbit tahun 2008
Akan Manusia. Medan : Cahaya.
terdapat kutipan berikut. “Anak, pada
Rahmini P. 2008. Pendidikan Anak
hakikatnya, titipan Tuhan. Sebagai
Jalanan. Surabaya : Bintang.
pemegang titipan, tidak selayaknya orang
Rohe, Siana. 2009. Bimbing Anak
tua menitipkan lagi sang anak kepada
Berakhlah Mulia. Surabaya: Gemerlap.
pembantu” (Nurudin, 2007)
D. Puspitasari, P. 2007. Memanusiakan
Jika Nunung mengutip pendapat Nurdin
Anak Manusia, Medan : Cahaya.
yang dikutip dalam buku Imron Zamani,
Rahmini P. 2008. Pendidikan Anak
penulisan kutipan yang benar adalah....
Jalanan. Surabaya : Bintang.
Rohe, S.2009. Bimbing Anak Berakhlak A. Nurudin (dalam Zamani, 2008 : 11)
Mulia. Surabaya : Gemerlap. menyatakan bahwa sebagai pemegang
titipan, tidak selayaknya orang tua

20
menitipkan lagi sang anak kepada Retorika Jalaludin Rosdakar Bandun
3 2006
pembantu. Modern Rakhmat ya g
B. Nurudin (2007) menyatakan bahwa Manakah penulisan daftar pustaka yang
sebagai pemegang titipan, tidak tepat untuk buku – buku di atas?
selayaknya orang tua menitipkan lagi A. Anas, Yusuf, 2006, Horizon Manusia,
sang anak kepada pembantu (dalam Jakarta; Al – Huda.
Zamani, 2008) Putrayasa, Ida Bagus, 2007, Kalimat
C. Menurut Nurudin dalam bukunya Efektif, Bandung; Rapika Aditama.
Zamani (2008) menyatakan bahwa Rakhmat, Jalaludin, 2006, Retorika
sebagai pemegang titipan, tidak Modern, Bandung; Rosdakarya.
selayaknya orang tua menitipkan lagi B. Anas, Yusuf. 2006. Horizon Manusia.
anak kepada pembantu. Jakarta, Al – Huda.
D. Nurudin (Imron Zamani, 2008 : 11) Putrayasa, Ida Bagus. 2007. Kalimat
menyatakan bahwa sebagai pemegang Efektif. Bandung, Rapika Aditama.
titipan, tidak selayaknya orang tua Rakhmat, Jalaludin. 2006. Retorika
menitipkan lagi anak kepada pembantu. Modern. Bandung, Rosdakarya.
E. Sebagai pemegang titipan, tidak C. Anas, Yusuf. 2006. Horizon Manusia.
selayaknya orang tua menitipkan lagi Jakarta: Al – Huda.
sang anak kepada pembantu (Nurudin, Putrayasa, Ida Bagus. 2007. Kalimat
2007) Efektif. Bandung: Rapika Aditama.
Rakhmat, Jalaludin. 2006. Retorika
12.Seseorang akan mengembangkan karangan Modern. Bandung: Rosdakarya.
yang bertema upaya meningkatkan D. Anas, Yusuf. 2006. Horizon Manusia.
kemampuan berwirausaha bagi remaja Jakarta: Al – Huda.
dengan kerangka karangan berikut : Putrayasa, Ida Bagus. 2007. Kalimat
1) Sarana bagi remaja untuk berwirausaha. Efektif. Bandung: Rapika Aditama.
2) Pentingnya meningkatkan kemampuan Rakhmat, Jalaludin. 2006. Retorika
berwirausaha. Modern. Bandung: Rosdakarya.
3) Langkah meningkatkan kemampuan E. Anas, Yusuf. 2006. “Horizon Manusia”.
berwirausaha Jakarta: Al – Huda.
4) Komponen yang mendukung Putrayasa, Ida Bagus. 2007. “Kalimat
kemampuan berwirausaha Efektif”. Bandung: Rapika Aditama.
5) Ketidakmampuan remaja berwirausaha Rakhmat, Jalaludin. 2006. “Retorika
Urutan kerangka yang paling tepat untuk Modern”. Bandung: Rosdakarya.
tema karangan di atas adalah....
A. 5, 1, 2, 4, 3 14.Identifikasi masalah:
B. 2, 3, 4, 5, 1 1. Apakah agar-agar itu?
C. 5, 2, 4, 3, 1 2. Bagaimana manfaat agar-agar bagi
D. 2, 4, 5, 1, 3 tubuh?
E. 4, 5, 2, 1, 3 Latar belakang karya ilmiah yang tepat
berdasarkan identitas masalah tersebut
adalah....
A. Agar-agar sangat disukai anak-anak,
karena rasanya yang manis. Agar-agar
yang paling disukai adalah yang
13.Berikut ini adalah identitas buku yang berbentuk jeli. Apalagi yang dikemas
digunakan sebagai sumber pustaka. dalam aneka warna.
Judul Penerbit B. Agar-agar ada yang berbahaya bagi
No Penulis Kota Tahun
Buku tubuh ada juga yang tidak. Untuk itu,
Horizon Yusuf Al – kita harus berhati-hati jika
1 Jakarta 2006
Manusia Anas Huda mengonsumsinya, supaya tubuh kita
Ida Rapika tidak keracunan.
Kalimat Bandun
2 Bagus Adiutam 2007 C. Jika dibandingkan dengan anak-anak,
Efektif g
Putrayasaa orang tua jarang sekali yang menyukai
agar-agar. Karena agar-agar kalau

21
dimakan tidak membuat perut kenyang,
sementara makanan lain bisa membuat
perut kenyang.
D. Agar-agar adalah makanan yang diolah
dari bahan dasar rumput laut, sangat
berguna bagi kesehatan tubuh kita,
terutama untuk membantu proses
pencernaan. Pencernaan akan menjadi
lancar jika sering mengkonsumsi agar-
agar.
E. Agar-agar sangat berbahaya bagi tubuh.
Untuk itu, kita harus berhati-hati jika
mengonsumsinya, supaya tubuh kita
tidak keracunan.

15.Objek wisata yang menawan dan


dilengkapi dengan atraksi budaya daerah,
banyak menentukan keberhasilan
pemasaran objek wisata tersebut.
Berdasarkan permasalah di atas, judul
karya tulis yang tepat adalah ...
A. Pemasaran Objek Wisata Perlu Terus
Ditingkatkan
B. Objek Wisata yang Menawan banyak
Menarik Wisatawan
C. Pemasaran Objek Wisata dan Seni
Budaya
D. Keberhasilan Pemasaran Objek Wisata
dan Seni Budaya
E. Peningkatan Pemasaran Objek Wisata
dan Seni Budaya

IND-6 KALIMAT EFEKTIF


KELAS XI

1. Kalimat di bawah ini tidak efektif karena


ketidaklogisan...
A. Ani menceritakan buku cerita untuk
adiknya.
B. Indonesia merupakan negara yang
berbentuk republik.
C. Perekonomian Indonesia mulai stabil
pascapandemi.

22
D. Perlombaan itu berhasil dimenangkan D. Pertemuan itu berhasil menghasilkan
oleh atlet asal Maluku Utara. lima keputusan penting.
E. Medan merupakan Ibukota Provinsi E. Ia selalu terbangun pada malam hari.
Sumatera Utara.
6. Kalimat berikut yang terhindar dari
2. Kalimat berikut manakah yang tergolong pleonasme adalah...
kalimat logis... A. Bagi para siswa-siswa diharapkan
A. Bima merupakan dosen tertampan di datang.
kampus itu. B. Penebangan pohon itu kami dibantu
B. Predikat wanita terpintar dimenangi oleh semua warga desa lainnya.
oleh Yulia. C. Saya hanya memiliki satu tanaman
C. Obat itu dapat menghilangkan kaki peliharaan saja
kesemutan. D. Siswa kelas IV senang memlihara
D. Kuda mampu melahap separuh ladang hewan dan tumbuhan
itu. E. Anda dilarang tidak boleh merokok
E. Berita itu sudah menyebar luas di
kalangan mahasiswa 7. Menurut berita di surat kabar, korban
penyakit flu burung sudah mewabah sangat
3. Kakak menolong anak itu dengan banyak sekali. Perbaikan kalimat tersebut
dipapahnya ke pinggir jalan. Perbaikan agar menjadi kalimat efektif adalah...
kalimat di atas agar menjadi kalimat efektif A. Meletakkan kata mewabah setelah kata
adalah.... penyakit
A. Kakak diganti menjadi Adik B. Memindahkan kata sudah sebelum kata
B. Pinggir jalan diubah menjadi tengah sangat
jalan C. Menghilangkan kata sekali di akhir
C. Dipapahnya seharusnya memapahnya kalimat
D. Menolong menjadi ditolong D. Menambahkan kata sudah dan banyak
E. Menambah kata “cara” setelah kata sebelum kata korban
“dengan” E. Kata menurut dihilangkan

4. Kalimat berikut tergolong kalimat efektif, 8. Kalimat berikut bukan kalimat efektif
kecuali... karena ketidaksesuain ejaan, kecuali...
A. Dalam rapat itu, pembina memutuskan A. Pengajian akan dimulai pada Jum’at
Ali sebagai ketua OSIS. nanti.
B. Dalam menanggulangi sampah B. Ria menggunakan aksesoris secara
diperlukan rencana yang matang. berlebihan.
C. Rudi selalu melakukan kekerasan, C. Pemukiman yang dilakukan pemerintah
penganiayaan, dan mencuri di sekitar berjalan dengan lancar.
jalanan sepi itu. D. Riadi ke sekolah untuk melegalisir
D. Pada pertemuan itu, Direktur Utama PT ijazahnya.
Kurnia Abadi membahas upah pekerja E. Ali sudah menganalisa permasalah itu
yang terlalu rendah. dengan detail.
E. Rina berhasil mendapat juara pertama
pada ajang perlombaan itu. 9. Satu di antara kearifan lokal yang
menghasilkan dari pengalaman adaptasi
5. Kalimat berikut tidak efektif karena masyarakat dengan lingkungannya adalah
pemilihan kata yang salah, kecuali... konsep hutan larangan. Konsep tersebut
A. Ina menghadiahi uang tunai kepada merupakan pandangan yang bersumber
Laila karena keberaniannya. pada pengetahuan masyarakat dalam
B. Orang yang pintar itu berhasil mengelola lingkungan secara tradisional.
mendudukkan posisi tinggi di Melalui konsep hutan larangan, masyarakat
perusahaan itu. menerapkan norma pengendali sikap dan
C. Mina membantu orang tua itu perilaku hidup dalam mengelola hutan.
menyeberangkan jalan. Hutan dianggap sebagai habitat warisan
yang perlu dipertahankan. Oleh karena itu,

23
pada komunitas masyarakat tertentu serta tata krama, norma dan hukum
berkembang kearifan lokal yang ditujukan setempat akan berubah sesuai dengan
untuk menjaga kelestarian fungsi kebutuhan situasi sosial.
lingkungan hidup. Tantangan dalam suatu budaya menjadi
Kalimat pertama dalam paragraf tersebut umpan balik yang terjadi dalam jaringan
akan menjadi efektif apabila diperbaiki kehidupan suatu sistem sosial. Hal ini
dengan cara... menandakan sedang berlangsungnya
A. Mengubah kata menghasilkan menjadi autopoesis yang menandakan bahwa suatu
dihasilkan. sistem sosial dalam suatu budaya
B. Menghilangkan kata dari. mengatur dirinya sendiri, suatu tanda
C. Menambah kata banyak sebelum kata bahwa suatu masyarakat dapat dikatakan
kearifan. sebagai suatu sistem yang hidup. Dalam
D. Mengubah kata adalah menjadi menghadapi perubahan inilah kearifan
merupakan. lokal memainkan peran dan fungsinya.
E. Menghilangkan kata satu di. Perbaikn kalimat pertama paragraf ketiga
agar menjadi kalimat efektif adalah...
10. Kearifan lokal adalah identitas atau A. Tantangan dalam suatu budaya dapat
kepribadian budaya sebuah bangsa yang terjadi karena umpan balik yang
menyebabkan bangsa tersebut mampu terjadi dalam jaringan kehidupan suatu
menyerap bahkan mengolah kebudayaan sistem sosial.
yang berasal dari luar/bangsa lain menjadi B. Tantangan dalam suatu budaya dapat
watak dan kemampuan sendiri. Kearifan terjadi jika umpan balik yang terjadi
lokal juga merupakan ciri khas etika dan dalam jaringan kehidupan suatu sistem
nilai budaya dalam masyarakat lokal yang sosial.
diturunkan dari generasi ke generasi. C. Tantangan dalam suatu budaya dapat
Lebih lanjut kearifan lokal juga terjadi sehingga umpan balik yang
didefinisikan sebagai kemampuan terjadi dalam jaringan kehidupan suatu
beradaptasi, menata, dan menumbuhkan sistem sosial.
pengaruh alam serta budaya lain yang D. Tantangan dalam suatu budaya dapat
menjadi motor penggerak transformasi terjadi sedangkan umpan balik yang
dan penciptaan keanekaragaman budaya terjadi dalam jaringan kehidupan suatu
Indonesia yang luar biasa. Hal ini juga sistem sosial.
bisa menjadi suatu bentuk pengetahuan, E. Tantangan dalam suatu budaya dapat
kepercayaan, pemahaman atau persepsi terjadi bahkan umpan balik yang
beserta kebiasaan atau etika adat yang terjadi dalam jaringan kehidupan suatu
menjadi pedoman perilaku manusia dalam sistem sosial.
kehidupan ekologis dan sistemik. 11. Para siswa-siswa sedang belajar di kelas.
Nilai-nilai yang mengakar dalam suatu Kalimat di atas merupakan kalimat tidak
budaya jelas bukan objek material yang efektif, perbaikan yang tepat agar menjadi
konkret, tetapi cenderung menjadi kalimat efektif adalah...
semacam pedoman bagi perilaku A. Siswa-siswa sedang berbelajar di
manusia. Dalam pengertian itu, untuk kelas.
mempelajarinya kita harus B. Para siswa sedang belajar di kelas.
memperhatikan bagaimana manusia C. Para siswa sedang belajar di kelas-
bertindak dalam konteks lokal. Dalam kelas
keadaan normal, perilaku orang terungkap D. Para siswa-siswi sedang belajar di
dalam batas-batas norma, etiket, dan kelas.
hukum yang terkait dengan wilayah E. Para siswa sedang mempelajari
tertentu. Namun, dalam situasi tertentu di sesuatu di kelas.
mana budaya menghadapi tantangan dari
dalam atau dari luar, respons dalam 12. Kalimat berikut menggunakan ejaan yang
bentuk reaksi dapat terjadi. Tanggapan tepat sehingga tergolong kalimat efektif...
dan tantangan adalah cara normal untuk A. Bacalah kutipan berita tersebut dengan
melihat bagaimana perubahan terjadi saksama!
dalam budaya. Struktur dan nilai sosial,

24
B. Para atlit berlatih dengan sungguh- A. Kalimat kelima
sungguh. B. Kalimat keempat
C. Tak lama lagi ujian praktek akan C. Kalimat ketiga
dilaksanakan. D. Kalimat kedua
D. Ia pergi ke sekolah untuk melegaisir E. Kalimat pertama
ijazah.
E. Sebuah penelitian dilakukan untuk
menganalisa penyebab terjadinya
suatu bencana.
13. Kami turut prihatin kepada terjadinya
musibah di daerah itu.
Kalimat efektif hasil dari perbaikan
kalimat tersebut adalah...
A. Kami turut prihatin sehingga
terjadinya musibah di daerah itu.
B. Kami turut prihatin maka terjadinya
musibah di daerah itu.
C. Kami turut prihatin atas terjadinya
musibah di daerah itu.
D. Kami turut prihatin supaya terjadinya
musibah di daerah itu.
E. Kami turut prihatin karena terjadinya
musibah di daerah itu.

14. Berikut adalah contoh kalimat efektif...


A. Mobil baru berangkat.
B. Silakan anda maju ke depan.
C. Saya melihat dengan mata kepala saya
sendiri.
D. Rasi adik Pak Amir sedang sakit.
E. Mahasiswa yang terkenal itu
menerima hadiah.

15. (1) Stroke atau Cerebrovascular Accident


(CVA) adalah kehilangan fungsi otak
yang diakibatkan oleh berhentinya suplai
darah ke bagian otak secara mendadak
(dalam beberapa detik) atau secara cepat
(dalam beberapa jam). (2) Kemudian,
pada otak timbul gejala dan tanda sesuai
dengan daerah yang terganggu, sehingga
dapat mengakibatkan kematian dan
penyebab utama kecacatan. (3) Stroke
merupakan salah satu penyakit yang dapat
mengakibatkan kematian dan penyebab
utama kecacatan tidak hanya pada
penderita di usia tua, tetapi juga di usia
muda. (4)Dalam pola kematian penderita IND-7 DAFTAR PUSTAKA, TABEL, DAN
rawat inap, stroke menduduki urutan GARFIK
pertama. (5) Akan tetapi, dari seluruh
penyebab kematian, stroke menduduki 1. Judul buku : Manajemen Personalia;
urutan ketiga terbesar setelah penyakit Penulis : Heidjrachman Ranupandoyo
jantung dan kanker. dan Suad Husnan;
Dari paragraf bacaan di atas, manakah Penerbit : BPFE Yogyakarta;
yang bukan merupakan kalimat efektif? Tahun terbit : 1989.

25
Penulisan daftar pustaka dari data buku B. Fransisca Wungu Prasasti.
tersebut di atas ialah ... 2008. Sayuran Hidroponik di Halaman
A. Heidjrachman Ranupandoyo dan Saud Rumah. Jakarta: Gramedia.
Husnan. 1989. Manajemen Personalia. C. Prasasti, Fransisca Wungu. 2008.
Yogyakarta: BPFE. Sayuran Hidroponik di Halaman
B. Ranupandoyo, Heidjrachman dan Rumah. Jakarta: Gramedia.
Husnan, Suad. 1989. Manajemen D. Prasasti, Fransisca Wungu.
Personalia. Yogyakarta : BPFE. 2008. Sayuran Hidroponik di Halaman
C. Ranupandoyo, Heidjrachman dan Suad Rumah. Jakarta: Gramedia.
Husnan. 1989. Manajemen Personalia. E. Prasasti, Fransisca Wungu. 2008.
Yogyakarta: BPFE. “Sayuran Hidroponik di Halaman
D. Ranupandoyo, Heidjrachman dan Suad Rumah.” Jakarta: Gramedia.
Husnan, Manajemen Personalia,
(Yogyakarta: BPFE), 1989. Perhatikan grafik berikut untuk
E. Heidjrachman Ranupandoyo dan Suad menjawab soal no.4 dan 5!
Husnan, Manajemen
Personalia (Yogyakarta: BPFE), 1989.

2. Judul : Agar Duit


Berkembang Biak
Penulis : Willam Tanuwijaya
Penerbit : Media Presindo, Yogyakarta
2006\
Penulisan daftar pustaka untuk buku diatas
ialah.... \
A. Tanuwijaya Willam, 2006, Agar Duit 4. Berdasarkan grafik di atas tentang tingkat
Berkembang Biak, Yogyakarta, Media kepuasan pelanggan terhadap pelayanan
Presindo rumah sakit X, manakan pernyataan berikut
B. Tanuwijaya, William. 2006. Agar Duit yang benar
Berkembang Biak. Yogyakarta: Media A. Tingkat kepuasan pelanggan terhadap
Presindo. pelayanan rumah sakit X cenderung
C. Willam Tanuwijaya, 2006, Agar Duit stabil pada Januari—Juli.
Berkembang Biak, Yogyakarta: Media B. Terjadi penurunan yang signifikan
Presindo. terhadap tingkat kepuasan pelanggan
D. Tanuwijaya, William. Agar Duit pada bulan November—Desember.
Berkembang biak. Yogyakarta: Media C. Kenaikan tertinggi kedua terjadi pada
Presindo.2006. bulan Februari—Maret.
E. William Tanuwijaya. Agar Duit D. Kenaiakn signifikan terjadi pada bulan
Berkembang biak. Yogyakarta. Media Agustus—September.
Presindo. 2006. E. Tingkat kepusan pelanggan cenderung
menurun pada bulan Agustus—
3. Judul : Sayuran Hidroponik di Desember.
Halaman Rumah
Pengarang : Fransisca Wungu Prasasti 5. Simpulan yang tepat berdasarkan grafik
Penerbit : Gramedia tersebut adalah….
Tahun terbit : 2008 A. Tingkat kepuasa pelanggan terhadap
Kota terbit : Jakarta pelayanan rumah sakit X cenderung
Bila Lina hendak mencantumkan identitas fluktuasi.
buku tempat ia mengutip pada daftar B. Tingkat kepuasa pelanggan terhadap
pustaka, penulisan daftar pustaka yang pelayanan rumah sakit X cenderung
benar adalah… menurun.
A. Fransisca Wungu Prasasti. 2008. C. Tingkat kepuasa pelanggan terhadap
Sayuran Hidroponik di Halaman pelayanan rumah sakit X cenderung
Rumah. Jakarta: Gramedia. menaik.

26
D. Tingkat kepuasa pelanggan terhadap
pelayanan rumah sakit X cenderung
stabil.
E. Terjadi penurunan signifikan pada 8. Sebuah pabrik di kota X mendata para
tingkat kepuasan pelanggan terhadap buruh berdasarkan jenis kelamin dan
pelayanan rumah sakit X dalam satu tingkat pendidikan.
tahun. Pernyataan tidak sesuai dari tabel tersebut,
kecuali….
Perhatikan table berikut untuk menjawab A. Buruh yang bekerja di pabrik tersebut
soal no.6 dan 7! paling banyak adalah tamatan SMP.
B. Lebih dari 8000 buruh berjenis kelamin
perempuan.
C. Jumlah tamatan SLTA laki-laki lebih
tinggi daripada jumlah tamatan SMP
perempuan.
D. Jumlah tamatan SD perempuan lebih
banyak daripada laki-laki.
E. Jumlah buruh tamatan SMP terbanyak
setelah SD.
6. Pernyataan yang sesuai berdasarkan grafik
tersebut, kecuali.... 9. Simpulan yang tepat berdasarkan tabel
A. Peningkatan suhu permukaan bumi tersebut ….
tertinggi terjadi tahun 2016 dan 2020. A. Buruh di pabrik tersebut paling banyak
B. Peningkatan suhu bumi tertinggi tamatan SD dan berjenis kelamin
keempat terjadi tahun 2017. perempuan.
C. Terjadi penurunan pada suhu B. Semakin tinggi tingkat pendidikan
permukaan bumi pada 2021. semakin banyak jumlah buruh di pabrik
D. Tahun 2015 terjadi peningkatan suhu tersebut.
bumu. C. Lebih dari setengah buruh di pabrik
E. Penurun signifikan terjadi di tahun tersebut tamatan SD.
2021. D. Buruh tamatan SLTA lebih diutamakan
bekerja di pabrik tersebut.
7. Simpulan yang tepat terkait grafik tersebut E. Buruh usia sekolah diperbolehkan
adalah bekerja di pabrik tersebut.
A. Suhu permukaan bumi cenderung
meningkat dari tahun 2011—2021. 10. Perhatikan diagram lingkaran berikut!
B. Suhu permukaan bumi cenderung
menurun dari tahun 2011—2021.
C. Suhu permukaan bumi cenderung stabil
dari tahun 2011—2021.
D. Suhu permukaan bumi cenderung
stagnan dari tahun 2011—2021.
E. Suhu permukaan bumi cenderung
fluktuasi dari tahun 2011—2021. Simpulam yang tepat berdasarkan
diagram tersebut adalah
A. Lebih dari setengah jumlah siswa yang
mendapat nilai di atas 80.
B. Kurang dari setengah total siswa
mendapat nilai di bawah 80.
Perhatikan tabel di atas untuk menjawab C. Lebih dari seperempat jumlah siswa
soal nomor 8 dan 9! yang mendapat nilai di bawah 65.
D. Setengah dari jumlah siswa mendapat
nilai di atas 80.

27
E. Lebih banyak siswa yang mendapat C. Lebih dari 20 ton hasil tangkapan
nilai tinggi daripada yang mendapat nelayan naik pada bulan Mei.
nilai rendah. D. Penurunan hasil tangkapan nelayan
mencapai 10 ton pada Februari.
11. Penulisan daftar pustaka yang tepat untuk E. Hasil tangkapan nelayan cenderung
buku berjudul Komposisi, karangan stabil.
Gorys Keraf, diterbitkan oleh Nusa Indah,
di Ende, Flores, tahun 1985 ialah ... 14. Perhatikan grafik berikut untuk
A. Keraf, Gorys. 1985. Komposisi. Ende, menjawab soal nomor 4 dan 5!
Flores: Nusa Indah.
B. Keraf, Gorys. 1985. Komposisi. (Ende,
Flores: Nusa Indah)
C. Gorys Keraf. 1985. Komposisi. Ende,
Flores: Nusa Indah.
D. Gorys Keraf. 1985. Komposisi. (Ende,
Flores: Nusa Indah)
E. Keraf, Gorys, Komposisi, (Ende,
Flores: Nusa Indah), 1985.

12. Judul : Membina Remaja Berapa persenkah siswa yang mengikuti


Pengarang : J.S. Badudu kegiatan ekstrakurikuler drama?
Penerbit : Pustaka Prima A. 20%
Tahun terbit : 2000 B. 15%
Kota terbit : Bandung C. 25%
Penulisan daftar pustaka yang benar D. 12%
adalah…. E. 13%
A. Badudu, J.S. 2000. Membina Remaja.
Bandung: Pustaka Prima. 15. Berdasarkan diagram lingkaran tersebut,
B. J.S.Badudu. 2000. Membina Remaja. jika total siswa SMA IP Azdkia sebanyak
Bandung: Pustaka Prima. 800 siswa, berapakan siswa yang
C. J.S.Badudu. 2000. Membina Remaja. mengambil kegiatan ekstrakurikuler tari?
Bandung: Pustaka Prima. A. 75 orang
D. Badudu, J.S. 2000. Membina Remaja. B. 63 orang
Pustaka Prima: Bandung C. 90 orang
E. Badudu, J.S. 2000. Membina Remaja. D. 86 orang
Bandung: Pustaka Prima. E. 96 orang
13. Perhatikan grafik berikut!

Pernyataan yang benar terkait grafik


tersebut adalah…. IND-8 AFIKSASI
A. Kenaikan yang cukup berarti dari KELAS XI
hasil tangkapan nelayan hanya terjadi
pada bulan Maret. Perhatikan bacaan berikut untuk
B. Pada bulan Juni hasil tangkapan menjawab soal nomor 1 s.d. 3!
nelayan naik sebnayak 10 ton.

28
1) Belakangan ini sampah menjadi konsen 5. Bentukan kata yang salah terdapat pada
besar dunia karena permasalahan yang A. Kalimat 1
terus ditimbulkannya. 2) Ada banyak B. Kalimat 4
kerugian yang disebabkan oleh sampah C. Kalimat 3
yang berdampak bagi kesehatan manusia. D. Kalimat 2
3) Nampaknya masih banyak orang yang E. Semua jawaban salah
enggan mendaur ulang sampah sehingga
menyebabkan sampah terus menumpuk. 4) Bacalah teks berikut untuk menjawab soal
Karena itu penelitian mengenai nomor 6 s.d. 7!
pemanfaatan kembali sampah penting Jantung merupakan salah satu organ yang
untuk dilakukan. sangat vital didalam tubuh manusia.
1. Berdasarkan teks di atas, manakah kalimat Karena itu, banyak orang yang berusaha
yang tidak mengandung kata beralomorf.... untuk selalu menjaga kesehatan
A. Kalimat 1 jantungnya. Akan tetapi, nampaknya
B. Kalimat 3 masih banyak orang yang kurang peduli.
C. Kalimat 2 Terbukti dari banyaknya korban jiwa
D. Kalimat 4 akibat penyakit-penyakit yang menyerang
E. Semua jawaban salah jantungnya. Untuk itu, penelitian
mengenai menjaga kesehatan jantung
2. Kata tidak baku terdapat pada kalimat perlu dilakukan.
berikut, kecuali.... 6. Bentukan kata yang salah terdapat pada
A. Kalimat 3 kalimat
B. Kalimat 2 A. Kalimat 1 dan 3
C. Kalimat 1 B. Kalimat 5 dan 3
D. Kalimat 1 dan 2 C. Kalimat 4 dan 2
E. Kalimat 2 dan 3 D. Kalimat 1 dan 3
E. Kalimat 4 dan 3
3. Proses pembentukan kata pemanfaatan
pada kalimat 4 bacaan di atas sama dengan 7. Imbuhan pada kata berikut menandai verba
proses pembentukan pada kata berikut, intransitif, kecuali
kecuali A. Menyerang D. Merupakan
A. Penelitian B. Menjadi E. Mendalam
B. Berguguran C. Mengenai
C. Berpakain
D. Kecamatan Bacalah teks berikut untuk menjawab soal
E. Permukiman nomor 8 s.d. 10!
Internet saat ini sudah sangat melekat
Bacalah teks berikut untuk menjawab soal dalam kehidupan sehari-hari, terutama
nomor 4 s.d. 5! dikalangan remaja. Memang, internet
Sampah yang banyak dihasilkan oleh cukup membantu dalam menyelesaikan
masyarakat sebenarnya masih bisa banyak pekerjaan. Akan tetapi, ternyata
digunakan agar tidak terlalu banyak internetpun tidak terlepas dari hal-hal
bertumpuk. Pemanfaatan kembali sampah negatif yang dapat berdampak buruk. Apa
pada dasarnya tidak memerlukan waktu lagi usia remaja merupakan saat-saat labil
lama. Yang terpenting adalah kreatifitas sehingga masih sering terseret arus. Untuk
dan ketekunan. Misalnya, sampah organik itu, penelitian mengenai bahaya internet
bisa diolah kembali menjadi pupuk. yang mampu merubah remaja penting
4. Kata berimbuhan yang salah digunakan untuk dilakukan.
terdapat pada 8. Bentukan kata berikut tidak benar dalam
A. Pemanfaatan bacaan di atas adalah, kecuali
B. Bertumpuk A. Apa lagi D.
C. Kreatifitas Dikalangan
D. Ketekunan B. Internet pun E. Terseret
E. Memerlukan C. Merubah

29
9. Kalimat bercetak miring dalam bacaan di C. Hasil dari letusan gunung berapi
atas tidak tergolong alomorf, kecuali tersebut membawa partikel-partikel
A. Kalimat 3 D. Kalimat 1 logam yang berbahaya.
B. Kalimat 5 E. Kalimat 4 D. Akan tetapi, penyebab utama
C. Kalimat 2 pencemaran udara di Indonesia
biasanya terjadi akibat polusi
10. Berdasarkan bacaan di atas, kata tidak kendaraan bermotor.
baku terdapat pada kata E. Kandungan-kandungan yang berbahaya
A. Merubah dalam rokok dapat menyebabkan
B. Labil gangguan pada kesehatan.
C. Terseret
D. Internet 15.Penangkapan ikan dengan menggunakan
E. Pekerjaan bahan peledak memang telah dilarang.
Akan tetapi, tetap saja ada oknum-oknum
Bacalah teks berikut untuk menjawab soal nakal yang masih menggunakan cara
nomor 1 s.d. 3! tersebut. Penangkapan dengan bahan
Internet dikalangan remaja saat ini tampaknya peledak memang dinilai efektif. Tidak
sudah menjadi kebutuhan pokok. Akan tetapi, akan memakan waktu yang lama dan juga
tidak semua remaja memanfaatkan internet akan memeroleh hasil tangkapan yang
dengan benar. Ada banyak remaja yang melimpah. Namun, hal tersebut
kecenderungan menyalahgunakan internet merupakan salah satu penyebab dari
sehingga berdampak buruk bagi dirinya dan pencemaran air.
lingkungannya. Bentukan kata yang salah terdapat pada
11. Proses pembentukan kata A. Kalimat 5
menyalahgunakan sama dengan proses B. Kalimat 3
pembentukan pada kata C. Kalimat 1
A. Pendayagunaan D. Kalimat 4
B. Menandatangani E. Kalimat 2
C. Mengambilalihkan
D. Pengalih bahasa
E. Keikutsertaan

12.Bentukan kata yang salah digunakan


terdapat pada
A. Kalimat 1
B. Kalimat 3
C. Kalimat 2
D. Kaliamt 1 dan 3
E. Kaliamt 2 dan 3

13.Bentukan kata yang salah terdapat pada


kalimat
A. Kalimat 1
B. Kalimat 3
C. Kalimat 2
D. Kaliamt 1 dan 3
E. Kaliamt 2 dan 3

14.Bentukan kata yang salah terdapat pada


kalimat
A. Pencemaran udara bisa juga disebabkan
oleh kejadian alam misalnya gunung
meletus.
B. Pencemaran udara tidak selalu
dikarenakan ulah manusia. IND-9 RESENSI

30
KELAS XI A. Sutradara Nicholas Frets cukup
berhasil membawa penonton untuk
1. Yang bukan dimaksud dengan terlibat di kehidupan sosial di zaman
kepengarangan dalam resensi buku adalah kemerdekaan dengan adegan-adegan
…. romatis maupun heroik yang
A. Tentang diri pengarang buku dimainkan oleh Aryo Bayu, dkk.
B. Perjalanan cinta pengarang B. Tema film ini cukup menginspirasi
C. Riwayat hidup pengarang penonton namun sayang pemilihan
D. Karya-karyanya yang lain tokoh (casting) tidak sesuai dengan
E. Biografi singkat pengarang karakter yang diberikan.
C. Meskipun terkenal karena
2. Berikut merupakan langkah-langkah kelihaiannya dalam memainkan
dalam membuat resensi film, kecuali …. peran antagonis untuk semua genre
A. Mengenali film yang akan diresensi film, karakterisasi “Dasimah” yang
B. Menandai bagian-bagian yang akan diperankan oleh Tri Hastuti dalam
dijadikan sebagai kutipan dalam Film Kembang Kuburan dipastikan
resensi gagal total.
C. Menonton film yang akan diresensi D. Film ini menjelaskan tentang usaha
dengan cermat seorang remaja dalam melawan
D. Membuat ikhtisar dari film yang Trypophobia, penyakit psikologi
akan diresensi aneh yang belum ada diagnosis
E. Menonton trailer film yang akan ilmiahnya dari dunia psikologi
diresensi E. Film yang disutradarai oleh Steve
Jericho ini gagal untuk meraih
3. Berikut ini adalah bahasa yang logis, penghargaan yang keduakalinya pada
tetapi kritis untuk meresensi sebuah novel Festival Film Bandung yang
adalah diselenggarakan tahun ini
A. Pengarang novel terlalu sembrono 5. Perhatikan penggalan isi resensi di
memberikan kiasan atau julukan bawah ini!
untuk para tokoh yang ada pada Penonton berhasil dibuat terkesan dengan
novel sehingga bisa menimbulkan pencahayaan (lighting) dan tata panggung
ketersinggungan. yang menstimulasi setiap tokoh untuk
B. Novel ini pada hakikatnya memainkan perannya secara maksimal
mengajarkan kita tentang arti namun ketidakjelian sang narator untuk
persahabatan, tetapi penggunaan mendramatisasi bagian klimaks
bahasa sarkas (kasar) sebaiknya tidak menjadikan ending episode terakhir ini
diperlihatkan secara terbuka. kurang impresif. Kalimat di atas
C. Novel ini sangat tidak layak untuk menunjukan sebuah ……
diterbitkan karena pengarangnya A. Tanggapan logis untuk sebuah opera
tidak memperhatikan pedoman- B. Kritik lugas untuk sebuah film
pedoman kesusastraan. C. Tanggapan untuk sebuah pementasan
D. Tema pada novel sangat D. Pujian dari sisi teknik untuk sebuah
menginspirasi kaum milenial untuk lakon cerita
memulai bisnis, mengambil risiko, E. Kritik untuk sebuah pementasan
dan mengatur strategi untuk drama
berbisnis yang positif. 6. Kalimat di bawah ini yang menunjukan
E. Pengaluran (plotting) pada novel kekurangan dari sebuah karya yang
terlalu monoton sehingga pembaca diresensi adalah ….
pastinya akan dibuat mengantuk A. Di luar film pertamanya The
serta ogah-ogahan membaca novel Sharkadon yang gagal total karena
tersebut. plot dan efek visual yang buruk, film
kedua yang masih bergenre fiksi
4. Kalimat di bawah ini mengungkapkan ilmiah The Earthquakes betul-betul
kelebihan sebuah film dalam sebuah memperlihatkan kualitas Jack Miller
resensi adalah …. sutradara kawakan.

31
B. Patung ini terlalu abstrak untuk untuk perang Asia Timur Raya.
dinikmati pencinta seni pemula Unsur resensi yang menonjol dalam
namun patung yang dibuat oleh I penggalan tersebut adalah ….
Nyoman Sukerta ini layak A. Penokohan
dinobatkan sebagai karya seni B. Sudut pandang
terhebat di tahun ini. C. Alur
C. Ponsel ini dilengkapi fitur-fitur D. Tema
canggih yang tidak dapat ditemukan E. Latar
di ponsel lainnya lagipula desain
yang keren dan tampak gaul 9. Berikut merupakan kalimat yang
membuat ponsel ini bisa diterima mengungkapkan kalimat yang
anak muda. mengungkapkan kelemahan buku….
D. Motor keluaran terbaru dari A. Buku ini sangat menarik, banyak
Kawasaki, ZX 250 RR adalah motor sekali gambar-gambar yang menarik.
tercepat di kelasnya meskipun Cerita yang disampaikan mudah
harganya relatif cukup mahal untuk untuk dimengerti.
kalangan orang Indonesia. B. Penulis sangat enjoy menggunakan
E. Buku ini kurang mencerminkan kata-kata yang digunakan dalam
pandangan politik Prabowo Subianto membuat novel sehingga memberi
secara utuh karena penulis biografi kesan tidak menggurui
tampak terpenjara oleh pola pikir C. Kemunculan buku ini telah
oposisi yang selalu kontradiktif memperkaya pengetahuan bisnis
dengan Sang Jenderal. khususnya untuk para pembisnis.
D. Kekuatan novel ini tidak hanya dari
7. Pernyataan yang dikutip dari resensi buku covernya saja yang menarik, isi
nonfiksi adalah … perlembar dari novel ini pun sangat
A. Novel ini termasuk novel serius yang menarik sehingga orang yang
tidak cukup menghibur seperti membacanya tidak mudah bosan
halnya dalam novel pop. E. Sayang sekali banyak sekali kata-
B. Mochtar Lubis memperlihatkan sikap kata yang sulit sehingga para
jiwa yang ditemukan pada diri orang- pembaca sulit mengartikan kata.
orang masa revolusi melalui tokoh
Hasil dan Guru Isa. 10. Berikut bukan merupakan langkah-langkah
C. Kemunculan buku ini telah menulis resensi ….
memperkaya khasanah literatur A. Menentukan jenis buku yang ingin
bisnis khususnya bagi pengusaha diresensi
konglomerat. B. Menentukan bagian paling menarik
D. Tidak berlebihan pula jika dikatakan dari buku
bahwa buku ini sangat baik dibaca C. Menyusun iktisar dari buku tersebut
oleh orang yang terusik oleh D. Menaksir harga buku
tamunya. E. Membaca buku hingga halaman
E. Kekuatan novel ini tidak hanya terakhir
terletak pada realitas hidup tokoh
Ongga yang sulit diduga. 11. Bila dicermati, buku ini lebih mirip buku
sejarah yang dikemas dalam bentuk yang
8. Pengarang menceritakan segala perasaan kocak, karikatural, menggelitik ditambah
dan pengalamannya semasa kecil. pengalaman menyaksikan sendiri
Pengarang berhasil memaparkan peperangan, jadilah buku ini sebagai
peristiwa secara berkesinambungan dan pengetahuan sejarah yang tidak akan
menuntut pembaca untuk menyelesaikan ditemukan dalam buku sejarah.
jalan ceritanya sampai akhir. Pengarang Penggalan resensi di atas merupakan
menceritakan pertemuan tentara Jepang bagian ….
dengan tokoh di kebun belakang rumah. A. Pembuka resensi
Cerita beranjak dari propanganda Jepang B. Penutup resensi
yang mengajak rakyat Indonesia berjuang C. Latar belakang buku

32
D. Keunggulan buku C. Kelemahan isi buku
E. Kelemahan buku D. Sumber tulisan yang diresensi
E. Cara pengarang mengutarakan
12. Seperti pada karya-karyanya yang lain, gagasan
dalam novelnya Sekayu N.H. Dini juga
menggunakan gaya bahasa orang pertama, 15. Seorang penulis resensi harus mengetahui
yakni menjadi salah satu tokoh; aku. tujuan penulisan buku yang akan diulas.
Kutipan resensi tersebut, membicarakan Tujuan penulisan buku dapat diketahui
salah satu unsur intrinsik novel, yaitu … dari ….
A. Alur A. bagian kata pengantar
B. Plot B. bagian pendahuluan
C. Setting C. bagian pembahasan
D. Tokoh D. bagian penutup
E. Sudut pandang E. bagian kepustakaan

13. Pertemuan Dua Hati, karya N.H. Dini


terbitan PT Gramedia tahun 1986 dengan
tebal buku 85 halaman, menyuguhkan
gambar dan kata-kata yang memikat
pembaca, di samping unsur yang lain.
Sampul buku didominasi warna hijau
dengan latar belakang pepohonan.
Ilustrasi utama gambar sesosok wanita
yang berhadapan dengan seorang bocah
cilik. Jika dilihat sepintas, sepertinya
sampul buku menggambarkan kasih
sayang ibu terhadap anaknya.
Kutipan di atas termasuk ke dalam resensi
buku fiksi karena ….
A. Memberikan penilaian buku secara
subyektif
B. Menjelaskan biografi pengarang
C. Mengemukakan identitas buku
secara lengkap
D. Membahas penggunaan bahasa dan
pengarang
E. Mengungkapkan unsure intrinsiknya
saja

14. Di samping tulisan-tulisan yang mencoba


membeberkan konsepisme post-
modernisme dan pengaruhnya terhadap
kebudayaan Indonesia, juga ada tulisan
yang mencoba memprediksi masa depan
peradaban manusia setelah nilai-nilai
kebudayaan modern terdekonstruksi oleh
semangat post-modernisme. Sayang,
tulisan penting di Horison dan Kalam
tidak terangkum dalam buku
Postmodernisme dan Masa Depan
Peradaban.
Yang dititikberatkan oleh resensator
dalam kutipan buku tersebut adalah ….
A. Isi buku
B. Perwajahan buku

33
5. Konjungsi yang tidak dipakai pada
penggalan bacaan tersebut, kecuali
IND-10 KONJUNGSI PENJELAS A. Syarat, final, disjugtif
KELAS XI B. Akibat, pertentangan, tak bersyarat
C. Perbandingan, final, aditif
1) Majelis hakim Pengadilan Negeri (PN) D. Aditif, penerang, sebab
Jakarta Selatan menjatuhkan vonis hukuman E. Disjugtif, aditif, final
pidana mati kepada Fredy Sambo. 2) Vonis ini 6. Pernyataan berikut tidak benar terkait
karena ia dinilai terbukti melakukan aturan penulisan singkatan dan akronim,
pembunuhan berencana terhadap ajudannya kecuali
Brigadir Yosua Hutabarat dan tanpa hak A. Singkatan yang dianggap umum
melakukan perbuatan yang menyebabkan diakhiri dengan tanda titik (.),
sistem elektronik tidak berfungsi sebagaimana misalnya PT.
mestinya. 3) Sedangkan Putri divonis penjara B. Akronim merupakan gabungan huruf
20 tahun dan dinilai terbukti secara sah dan tiap kata yang diubah menjadi kata,
meyakinkan bersalah turut serta melakukan misalnya Puskesmas.
pembunuhan berencana terhadap Yosua. 4) C. M. Hum. merupakan contoh dari
Putusan tersebut dijatuhkan oleh ketua majelis akronim.
hakim Wahyu Iman Santoso dengan anggota D. Penulisan Rudal sudah benar menurut
Morgan Simanjuntak dan Alimin Ribut aturan penulisan akronim.
Sudjono. E. Semua penulisan singkatan harus
1. Berdasarkan kalimat di atas, manakah menggunakan huruf kapital.
kalimat yang mengandung konjungsi Pemilu 2024 memang masih cukup lama,
aditif tetapi tahapan penyelenggaraan pemilu sesuai
A. 1, 3, 4 Pasal 167 ayat (6) UU No.7/2017 dimulai
B. 2, 3, 4 paling lambat 20 bulan sebelum hari
C. 1, 4, 2 pemungutan suara. Sehingga, jika KPU
D. 1, 2, 3 mengusulkan pemungutan suara pemilu 2024
E. (semua jawaban salah) tanggal 21 Februari 2024, maka persiapan
2. Penggunaan konjungsi aditif dan tahapan pemilu 2024 harus dimulai sekitar
konjungsi sebab sekaligus digunakan bulan Juli 2022. Secara regulatif dan teknis,
pada kalimat penyelenggara pemilu saat ini memang
memiliki pengalaman pada pemilu 2019.
A. Satu Meskipun demikian, menurut Fritz Edward
B. Tiga Siregar, Anggota Bawaslu RI tetap saja
C. Empat dibutuhkan perbaikan untuk hal-hal yang
D. Dua selama ini dianggap kurang. Misalnya,
E. (semua salah kecuali opsi A) ketentuan-ketentuan dalam PKPU dan
3. Penggunaan konjungsi pertentangan dan Perbawaslu perlu dimatangkan serta
konjungsi aditif sekaligus terdapat pada disesuaikan dengan perkembangan zaman.
kalimat 7. Pernyataan yang sesuai terkait bacaan di
atas adalah
A. Satu A. Penulisan UU seharusnya
B. Tiga menggunakan huruf kecil karena
C. Empat dianggap singkatan umum.
D. Dua B. Penulisan pemilu seharusnya
E. (semua salah kecuali opsi C dan D) PEMILU.
4. Kalimat yang tidak menggunakan C. Penulisan Bawaslu dan Perbawaslu
konjungsi penerang terdapat pada kalimat menggunakan huruf kecil karena
A. Satu dan tiga merupakan kata umum.
B. Dua dan empat D. Masih terdapat kesalahan penulisan
C. Tiga dan dua pemilu.
D. Satu dan empat E. Penulisan PKPU tidak dibenarkan
E. (semua opsi salah) karena dianggap tidak umum.

34
8. Penulisan kalimat yang sesuai dengan bukanlah gratikasi itu telah diterima
EYD adalah … KPK.
A. “Sampah yang muncul pasca-banjir E. Dalam usaha menegakkan aturan dan
di DKI Jakarta, mengalami menghilangkan kesan tidak adil
peningkatan rata-rata 1.100 ton per Komisi Penyiaran Indonesia Daerah
hari dengan jumlah terbesar berasal JATENG akan menertibkan Radio
dari wilayah Jakarta Utara dan Siaran Pemerintah Daerah.
Jakarta Barat,” kata Kepala Dinas
DKI, Unu Nurdin. 10. Penulisan singkatan dan akronim berikut
B. “Sampah yang muncul pascabanjir di tidak salah, kecuali
Dki Jakarta, mengalami peningkatan A. Auri dinobatkan sebagai angkatan
rata-rata 1.100 ton per hari dengan bersenjata terbaik se-Asia Tenggara.
jumlah terbesar berasal dari wilayah B. Layanan pemeriksaan gigi sudah
Jakarta Utara dan Jakarta Barat,” tersedia di puskesmas terdekat.
kata Kepala Dinas Dki, Unu Nurdin. C. Radit dipilih untuk membacakan
C. “Sampah yang muncul pasca banjir UUD 1945 saat upacara nanti.
di DKI Jakarta, mengalami D. Dr. Ningrum, MSC. adalah dosenku
peningkatan rata-rata 1.100 ton saat kuliah di UGM.
perhari dengan jumlah terbesar E. Kaina adalah salah satu mahasiswa
berasal dari wilayah Jakarta Utara penerima beasiswa UNPAD.
dan Jakarta Barat,” kata Kepala
Dinas DKI, Unu Nurdin. Perhatikan bacaan berikut!
D. “Sampah yang muncul pasca-banjir 1) Pada momen perayaan Hari Kanker
di DKI Jakarta, mengalami Sedunia tahun 2023, telah menyadarkan kita
peningkatan rata-rata 1.100 ton per bahwa kanker masih menjadi salah satu
hari dengan jumlah terbesar berasal penyakit dengan angka kematian tertinggi di
dari Wilayah Jakarta Utara dan Indonesia. 2) Saat ini kemenkes menargetkan
Jakarta Barat,” kata Kepala Dinas akses pelayanan kanker merata di seluruh
DKI, Unu Nurdin. Indonesia. 3) Diharapkan pelayanan kanker di
E. “Sampah yang muncul pascabanjir di daerah terpencil bisa sama dengan yang ada di
DKI Jakarta, mengalami peningkatan perkotaan. 4) Kementerian Kesehatan telah
rata-rata 1.100 ton per hari dengan menjalin kerja sama dengan University of
jumlah terbesar berasal dari wilayah Texas MD Anderson Cancer Center dan
Jakarta Utara dan Jakarta Barat,” melakukan kunjungan ke RS Ngoerah serta
kata Kepala Dinas DKI, Unu Nurdin. Rsud Bali Mandara sebagai bentuk upaya
9. Penulisan huruf kapital yang mengikuti melakukan percepatan terhadap target tersebut.
EYD terdapat dalam kalimat berikut … 5) dr. R. Soeko Werdi Nindito, MARS
A. Senin malam banjir lahar kembali selaku Direktur Utama RS Kanker Dharmais
terjadi di sungai Kali Putih dan mengatakan bahwa kerjasama dan
sempat menerobos area dekat pasar pendampingan yang dilakukan saat ini
Jumojoyo. merupakan upaya yang dilakukan agar
B. Setelah sejak lama diserahkan ke masyarakat di daerah terpencil dapat lebih
DPR RI pemerintah melalui cepat dan mudah untuk mendapatkan akses
mendagri akan memberikan yang sama seperti masyarakat di kota besar. 6)
penjelasan mengenai rancangan Nantinya, tidak hanya penanganan kanker
Undang-Undang Keistimewaan DIY yang dapat dilakukan, namun layanan jantung,
di hadapan Komisi II. stroke, ginjal, dan layanan ibu dan anak juga
C. Enam partai kecil di Salatiga akan dapat dilakukan. 7) Pemerintah menyediakan
menggetarkan Pemilihan Umum alat pemeriksaan kanker, pemerataan
Kepala Daerah (pemilukada) kota penyebaran alat kesehatan di 514 kabupaten
Salatiga yang digelar pada 8 Mei dan kota. 8) Beasiswa juga disediakan untuk
2013 mendatang. dokter umum dan dokter spesialis kanker. 9)
D. Sebagaimana diungkapkan oleh Namun, upaya yang lebih efektif dalam
Jassin surat dari PSSI yang berisi menemukan kanker pada stadium yang lebih
penegasan bahwa Piala AFF dini salah satunya dengan deteksi dini. Deteksi

35
dini seperti Sadanis (Pemeriksaan Payudara Cancer Center dan melakukan
Secara Klinis), Sadari (Pemeriksaan Payudara kunjungan ke rs Ngoerah serta rsud
Sendiri) atau IVA test (Inspeksi Visual Asam Bali Mandara sebagai bentuk upaya
Asetat) dapat meningkatkan peluang melakukan percepatan terhadap target
kesembuhan hingga 80--90%. (DJ) tersebut.
11. Menggunakan konjungsi apakah kalimat D. Kementerian Kesehatan telah
4 pada bacaan di atas? menjalin kerja sama dengan
University of Texas Md Anderson
A. Disjungtif Cancer Center dan melakukan
B. Final kunjungan ke R.S. Ngoerah serta
C. Pertentangan R.S.U.D. Bali Mandara sebagai
D. Syarat bentuk upaya melakukan percepatan
E. Aditif terhadap target tersebut.
12. Pada paragraf kedua bacaan di atas, E. Kementerian Kesehatan telah
konjungsi final digunakan pada kalimat menjalin kerja sama dengan
…. University of Texas MD Anderson
A. Lima Cancer Center dan melakukan
B. Enam kunjungan ke RS Ngoerah serta
C. Tujuh RSUD Bali Mandara sebagai bentuk
D. Delapan upaya melakukan percepatan terhadap
E. (semua opsi salah) target tersebut.
13. Pernyataan berikut BENAR terkait 15. Pernyataan berikut BENAR terkait
penggunaan konjungsi pada bacaan di bacaan di atas, kecuali ….
atas, kecuali …. A. Terdapat kesalahan penulisan
A. Terdapat kesalahan penggunaan singkatan pada kalimat 5 bacaan
konjungsi pada kalimat 6. tersebut.
B. Bacaan tersebut banyak B. Penulisan akronim pada kalimat 2
menggunakan konjungsi aditif. tidak mengikuti aturan EyD V.
C. Konjungsi pertentangan pada kalimat C. Penulisan Sadanis dan Sadari
9 dapat diganti dengan tetapi. seharusnya SADANIS dan SADARI.
D. Selain kata agar (kalimat 5) D. Penulisan Rsud seharusnya RSUD.
konjungsi final yang lain adalah kata E. Penulisan kemenkes seharusnya
untuk. Kemenkes.
E. Pada bacaan tersebut, tidak adala
kalimat yang menggunakan konjungsi
syarat.
14. Perbaikan kalimat 4 bacaan di atas adalah
A. Kementerian Kesehatan telah
menjalin kerja sama dengan
University of Texas MD Anderson
Cancer Center dan melakukan
kunjungan ke RS Ngoerah serta
RSUD Bali Mandara sebagai bentuk
upaya melakukan percepatan terhadap
target tersebut.
B. Kementerian kesehatan telah menjalin
kerja sama dengan University of
Texas MD Anderson Cancer Center
dan melakukan kunjungan ke R.S.
Ngoerah serta Rsud Bali Mandara
sebagai bentuk upaya melakukan
percepatan terhadap target tersebut.
C. Kementerian Kesehatan telah
menjalin kerja sama dengan
University of Texas MD Anderson

36
B. Kata ulang berimbuhan
C. Kata ulang dwilingga
D. Kata ulang sebagian
E. Kata ulang dwilingga salin suara

2. Kata ulang yang bentuk dasarnya berupa


kata berimbuhan adalah....
A. Orang-orangan
B. Beras-petas
C. Cumi-cumi
IND-11 REDUPLIKASI (KATA ULANG) D. Tanam-tanaman
KELAS XI E. Lelaki

1. (1) Menteri Pendidikan dan Kebudayaan 3. Ibu membeli mobil-mobilan untuk adik.
(Mendikbud), Nadiem Makarim Nosi reduplikasi pada kalimat tersebut
menyebutkan bahwa prioritas Merdeka sama dengan kalimat di bawah ini, yaitu...
Belajar 2021 akan berfokus pada beberapa A. Saudara-saudara saya pergi ke pantai.
hal. (2) Fokus pertama meliputi B. Kedua mobil itu lecet-lecet karena
pembiayaan pendidikan yang akan bersenggolan.
menyasar pemilik Kartu Indonesia Pintar C. Mereka makan-makan di sekolah.
(KIP) Kuliah dengan target 1,095 juta D. Adikku pandai membuat rumah-
mahasiswa dan KIP Sekolah dengan target rumahan dari pasir.
17,9 juta siswa. (3) “Kemudian, ada E. Di pesantren, ia mempelajari bahasa-
bantuan pemerintah kepada 13 SILN dan bahasa internasional.
2.236 lembaga,” ungkap Nadiem dalam
acara Taklimat Media Awal Tahun 2021. 4. Bentuk dasar pada kata ulang tumbuh-
(4) Dengan adanya hal itu, program- tumbuhan adalah tumbuh.
program yang dijalankan diharapkan dapat SEBAB
berjalan lancar. (5) Fokus kedua adalah Bentuk dasar pada kata ulang tersebut
program digitalisasi sekolah dan medium berupa kata berimbuhan.
pembelajaran melalui empat sistem
penguatan platform digital, dan delapan 5. Bacalah paragraf di bawah ini dengan
layanan terpadu Kemendikbud. (6) Fokus saksama!
ketiga adalah pembinaan peserta didik, Lingkungan fisik Indonesia yang terdiri
prestasi, talenta, dan penguatan karakter. atas daerah-daerah yang terbentang dari
(7) Prioritas ini akan diciptakan melalui Sabang sampai Merauke yang bertabur
tiga layanan pendampingan advokasi dan banyak beribu-ribu pulau di seluruh
sosialisasi penguatan karakter. (8) Selain Nusantara berpengaruh pada praktik sosial,
itu, prioritas ini diciptakan dengan beberapa anata lain berupa perilaku-perilaku
cara, seperti pembinaan peserta didik oleh masyarakat.
345 pemerintah daerah serta peningkatan Pernyataan di bawah ini yang tidak benar
prestasi, manajemen talenta kepada 13.505 mengenai penggunaan kata ulang dalam
pelajar, dan sebagainya. (9) Sementara itu, paragraf di atas, yaitu....
untuk fokus keempat, Kemendikbud A. Terdapat kata ulang dwilingga dalam
menargetkan adanya pendidikan kepada paragraf di atas.
19.624 guru penggerak tahun ini. (10) Lalu, B. Semua penggunaan kata ulang dalam
akan ada sertifikasi terhadap 10 ribu guru paragraf di atas sudah tepat.
dan tenaga kependidikan. (11) Selain itu, C. Terdapat kata ulang berimbuhan dalam
ada pula perekrutan guru PPPK oleh 548 paragraf di atas.
pemerintah daerah serta penjaminan mutu D. Jumlah kata ulang dwilingga dalam
sekolah penggerak dan organisasi paragraf di atas adalah dua kata ulang.
penggerak kepada 20.438 orang guru. E. Penggunaan kata ulang berimbuhan
Penggunaan kata ulang pada bacaan di atas dalam paragraf di atas tidak tepat.
termasuk jenis kata ulang ?
A. Kata ulang dwipurwa

37
6. Kata-kata di bawah ini yang tidak termasuk C. Siswa itu terus-menerus memanggil-
kata ulang adalah.... manggil gurunya untuk berdiskusi.
A. Tertawa-tawa, terngiang-ngiang D. Peristiwa-peristiwa yang dikarenakan
B. Putra-putri, kunang-kunang bentuk kedisiplinan harus terus saling
C. Bercubit-cubitan, berpandang- sambung-menyambung diterapkan.
pandangan E. Banyak jenis sayur-mayur yang
D. Tetamu, rerumputan dibutuhkan tubuh kita.
E. Bolak-balik, sayur-sayuran
Perhatikan bebrapa kata ulang di bawah ini
7. Penggunaan kata ulang yang sudah tepat untuk menjawab soal nomor 1 s.d. 3!
terdapat dalam kalimat … 1. Berlari-lari 6. Sayur-mayur
A. Serombongan pelukis-pelukis 2. Teka-teki 7. tetamu
mengadakan pameran di kota kami. 3. Menetes-netes 8. Tanam-tanaman
B. Dalam menghadapi kesulitan kita wajib 4. Buah-buahan 9. Kuat-kuat
tolong-menolong. 5. Mobil-mobilan 10. Terbatuk-batuk
C. Ibu sudah mengajar kami hal masak-
memasak sejak sekolah dasar. 11. Kata ulang dwilingga salin suara dan
D. Adik selalu menyebut-nyebut nama ibu dwipurwa terdapat pada nomor....
sewaktu ia sakit. A. 2,4,7
E. Pikirkan hal itu sangat masak-masak B. 5,8,7
supaya tidak menyesal akhirnya. C. 1,5,8
D. 2,6,7
8. Kata ulang yang digunakan secara tepat E. 3,6,8
terdapat dalam kalimat….
A. Semua mobil-mobil harus diparkir di 12.Kata ulang yang bermakna intensitas
tempat yang telah disediakan. kualitatif terdapat pada nomor....
B. Banyak guru-guru hadir dalam A. Tiga
pertemuan itu. B. Sembilan
C. Para murid-murid berbaris dengan rapi. C. Delapan
D. Pemuda-pemuda sibuk menghadapi D. Tujuh
peringatan hari Proklamasi. E. Satu
E. Segenap anggota-anggota Korpri wajib
apel. 13.Kata ulang berimbuhan terdapat pada
nomor....
9. Berhari-hari lamanya mereka berjalan A. 2,9,7
mondar-mandir keluar masuk kantor. B. 1,4,2
Dilihatnya orang bercakap-cakap dengan C. 8,10,6
serius. Tak seorang pun terdengar tanya- D. 3,4,9
menanya, tertawa-tawa, atau lihat-melihat. E. 4,5,10
Dari paragraf di atas ada dua kata ulang
yang bermakna resiprok, yaitu.... 14.Permukaan laut yang naik mengakibatkan garis
A. Berhari-hari, mondar-mandir pantai mulai terkikis dan menyebabkan daratan
B. Tanya-menanya, lihat-melihat pinggir pantai mulai tenggelam. Setidaknya, delapan
C. Bercakap-cakap, berhari-hari pulau dataran rendah di Samudra Pasifik telah lenyap
D. Tertawa-tawa, lihat-melihat di bawah permukaan laut, sementara beberapa di
E. Mondar-mandir, bercakap-cakap antaranya seperti kepulauan Maladewa (Maldives),
Fiji, dan Kiribati sampai sekarang masih berisiko
10. Kalimat yang menggunakan kata ulang tinggi tenggelam. Pengikisan garis pantai ini ikut
yang tepat adalah.... menempatkan banyak Kota-Kota Metropolitan
A. Karena semua penumpang saling tinggi populasi manusia dekat dataran pantai atau
dorong-mendorong, akhirnya mereka delta sungai (Shanghai, Bangkok, Jakarta, Tokyo,
jatuh. dan New York) dalam bahaya besar. Bahkan, hampir
B. Kita harus tolong-menolong sebagai setengah daratan Belanda sudah “tertelan” di bawah
teman. permukaan laut.

38
Perbaikan penulisan kata ulang dalam paragraf di atas
adalah....
A. Tidak perlu diperbaiki
B. banyak kota-kota Metropolitan
C. banyak kota Metropolitan
D. banyak kota metropolitan
E. banyak Kota Metropolitan

15.Mereka selalu tolong-menolong dalam


berbagai hal. Kata ulang yang memiliki
nosi sama dengan kata ulang di atas,
terdapat dalam kalimat...
A. Sebelum musim hujan, warga bahu-
mambahu membersihkan selokan.
B. Rumah-rumah penduduk yang dilanda
banjir rusak berat.
C. Pada hari Lebaran, kami bertandang ke
rumah tetangga-tetangga.
D. Kami menari-nari kegirangan karena
mendapat hadiah dari ayah kami.
E. Kami makan dengan lauk-pauk
seadanya.

39
2. Kalimat utama paragraf pertama pada
bacaan di atas adalah...
A. Usulan Gita Wirjawan.
B. Usulan mantan Menteri Perdagangan.
C. Mantan Menteri Perdagangan
mengusulkan mencetak uang baru.
D. Mantan Menteri Perdagangan
mengusulkan ke Bank Indonesia.
E. Bank Indonesia mencetak uang baru
Rp4000 triliun.

3. Ide pokok bacaan di atas adalah...


IND-12 IDE POKOK A. rencana mencetak uang baru
KELAS XI B. mencetak uang baru
C. Mantan Menteri Perdagangan dikritik
Bacalah paragraf di bawah ini untuk D. Menteri Perdagangan Gita Wirjawan
menjawab soal nomro 1 s.d. 3! E. usulan mencetak uang baru
Usulan mantan Menteri Perdagangan,
Gita Wirjawan, ke Bank Indonesia mencetak 4. Hampir sama dengan jaipong di pedalaman
uang baru Rp4000 triliun dikritik sejumlah Jawa Barat, ronggeng di sekitar Cirebon
pihak termasuk pengamat ekonomi, Arim dan Indramayu atau tayub di sekitar Blora
Nasim. Secara teori, inflasi terjadi ketika dan Cepu, gandrung juga menghadirkan
jumlah uang bertambah, tetapi tidak diikuti penari, penyanyi, musik pengiring, dan
dengan penambahan barang. Oleh karena itu, penari spontan yang berasal dari penonton
pencetakan uang baru senilai Rp4000 triliun secara bergantian. Bagaimanapun,
sementara pertumbuhan ekonomi stagnan gandrung tidak sama dengan seblang
diprediksi minus. Tidak hanya akan meskipun keduanya merupakan budaya
menimbulkan inflasi, tetapi juga hiperinflasi. masyarakat Osing. (Seblang, yang
Ungkap Arim kepada Mediaumat.news, merupakan tarian ungkapan rasa syukur
Minggu (3/5/2020). sesuai masa panen, masyarakatkan si penari
Arim pun mencontohkan hiperinflasi untuk kesurupan. Seblang dilaksanakan
yang terjadi di Zimbawe dan Indonesia di dalam situasi khusuk dan unsur sakralnya
masa Soekarno. Tahun 2002-2005, Zimbabwe kuat, sedangkan gandrung bersifat profan
mengalami hiperinflasi karena menyetak uang dan bisa ditampilkan setiap saat. Maka,
besar-besaran untuk membayar utangnya. gandrung mempunyai kesamaan dan
Begitu juga di Indonesia tahun 1963-1965 perbedaan dengan jenis tarian yang lain.
mencetak uang besar-besaran sehingga terjadi Apa gagasan utama paragraf di atas?
hiperinflasi dan akhirnya melakukan sanering A. Gandrung menghadirkan penari,
dengan pemotongan nilai uang 1000 persen; penyanyi, musik pengiring, dan penari
uang Rp1000 menjadi Rp1. ―Jadi, inflasi itu spontan.
disebabkan oleh pertambahan uang kertas (fiat B. Gandrung memiliki persamaan dan
money) yang tidak dibackup oleh pertumbuhan perbedaan dengan jenis tarian lain.
ekonomi atau peningkatan aset ril, tidak ada C. Gandrung hampir sama dengan jaipong,
hubungannya dengan tujuan pencetakan uang ronggeng, atau tayub.
apakah untuk foya-foya atau bukan,” D. Gandrung tidak sama dengan seblang,
bebernya. dan tari yang lain.
E. Gandrung tidak sama dengan seblang
1. Apa gagasan utama paragraf kedua pada meskipun sama-sama budaya Osing.
bacaan yang disajikan?
A. contoh inflasi yang pernah terjadi Bacalah paragraf berikut dengan
B. contoh hiperinflasi yang pernah terjadi saksama untuk menjawab nomor 5 dan
C. hiperinflasi yang terjadi di Indonesia 6!
D. hiperinflasi di masa Soekarno (1)Objek wisata Pangandaran menyediakan
E. hiperinflasi yang terjadi di Zimbawe transportasi rekreasi untuk memudahkan

40
wisatawan menikmati keindahan pantai. (2) untuk menggulirkan demokrasi yang
Wisatawan domestik maupun mancanegara berbiaya tinggi dan demokrasi harus
dapat menggunakan transportasi untuk menyiapkan peraturan dan undang-undang
menikmati keindahan alam. (3) Di yang melanggengkan para kapitalis untuk
sepanjang tepi Pantai Pangandaran terlihat secara bebas mengeruk kekayaan negeri ini.
berjejer perahu untuk disewakan. (4)
Dengan biaya Rp. 1.000,00 saja per orang, 7. Dari bacaan di atas, manakah simpulan
para wisatawan dapat berputar di sekitar yang paling tepat?
pantai dan menikmati keindahan taman A. Omnibus law bukan kepentingan rakyat.
laut. (5) Para pedagang ikut meramaikan B. RUU Cipta Kerja bukan kepentingan
situasi Pantai Pangandaran. cukong.
C. RUU Cipta Kerja kepentingan rakyat.
5. Kalimat utama paragraf di atas ditandai D. Omnibus law bukan kepentingan
dengan nomor .... kapitalis.
A. (2) E. Perjuangan Omnibus law.
B. (1)
C. (3) 8. Pertanyaan berikut merupakan pertanyaan
D. (5) yang jawabnnya tidak terdapat dalam
E. (4) bacaan, kecuali....
A. Kepentingan para cukong mana yang
6. Kalimat penjelas yang tidak mendukung diperjuangkan dalam omnibus law?
kalimat utama dalam paragraf tersebut B. Apakah yang dimaksud dengan
adalah nomor .... kapitalis?
A. (1) C. Apa kepanjangan RUU Cilaka?
B. (4) D. Mengapa kapitalisme dan demokrasi
C. (2) dikatakan dua sisi mata uang?
D. (5) E. Siapakah ketua Pusako dari Fakultas
E. (3) Hukum Universitas Andalas?

Bacalah paragraf berikut dengan cermat 9. Gerakan di dunia maya untuk


untuk menjawab soal nomor 7 dan 8! menyelamatkan hiu supaya tak menjadi
Bukan kepentingan nasional yang korban pandemi covid-19 makin
diperjuangkan dalam omnibus law, tetapi meningkat. Seruan ini muncul setelah
kepentingan para cukong. Omnibus Law sebelumnya pada September lalu,
RUU Cipta Kerja ‗cilaka‘ berbasis kelompok perlidungan hiu yang berbasis di
faradigma kapitalisme. Kapitalisme Los Angeles, Shark Allies, membuat
menempatkan pertumbuhan ekonomi di unggahan di Facebook dan membuat petisi
atas segala-galanya dan melupakan di Change.org mengenai bahaya yang
kesejahtraan rakyat. Kepentingan para dihadapi populasi hiu. Bahaya ancaman
kapitalis mendapatkan pelayanan terdepan. terhadap kelangsungan hiu ini di saat
Peneliti Pusat Studi Konstitusi (Pusako) negara-negara di dunia tengah berlomba
Fakultas Hukum Universitas Andalas, menemukan vaksin Corona. Seperti dikutip
Charles Simabura, mengatakan bahwa dari Phys, Sabtu (10/10/2020) hiu menjadi
Omnibus Law RUU Cipta Kerja hanya sumber bahan penting dalam pembuatan
menitik beratkan pada kepentingan vaksin yang dikenal sebagai squalene.
ekonomi. Menurut Charles, tidak ada Kalimat utama paragraf di atas terletak
pertimbangan keadilan dan kesejahteraan pada....
sosial dalam rancangan undang-undang A. kalimat kelima
tersebut. B. kalimat ketiga
Omnibus Law RUU ―Cilaka‖ C. kalimat pertama
dipastikan akan lolos dan goal menjadi UU. D. kalimat keempat
Institusi DPR dan partai politik hanyalah E. kalimat kedua
setempel dari kerakusan para kapitalis.
Kapitalisme dan demokrasi adalah dua sisi 10. Polemik kasus Bank Century berdampak
mata uang. Kapitalisme menyiapkan modal pada penurunan minat investor saham,

41
nilai transaksi di bursa semakin kecil, dan E. Di Indonesia dan di semua negara
indeks harga saham gabungan (IHSG) dengan sistem politik demokrasi saat ini
tertekan. Para investor lokal maupun luar makin tak tahu malu.
cenderung menahan diri dan bersikap
menunggu kepastian mengenai kasus 12. Corona telah memasuki fase yang sangat
Century yang sudah mengarah pada tanah mengkhawatirkan bagi negara-negara
politik. Akibatnya, indeks stagnan dunia. Wabah yang berawal di China pun
sementara transaksi harian turun. memasuki dunia Islam. Persoalan ini pun
Ide pokok-paragraf tersebut merata dan berdampak pada hilangnya
dikembangkan dengan gagasan penjelas nyawa dan ekonomi suatu negara. Corona
berikut, kecuali.... menjadi pukulan telak bagi dunia,
A. dampak polemik kasus Bank Century khususnya kegagalan kapitalisme global
B. tertekannya indeks saham gabungan dalam menjaga jiwa dan nyawa manusia.
C. kecilnya nilai transaksi Farid Wajdi, pemerhati politik
D. sikap para investor internasional, ketika dihubungi Media
E. stagnasi indeks Umat menjelaskan bahwa dalam
perhitungan ADB dampak Corona sekitar
11. Di Indonesia dan di semua negara dengan 77 hingga 347 miliar US dolar. Dampak
sistem politik demokrasi, membagi-bagi yang cukup besar di sisi ekonomi.
jabatan kepada parpol pendukung dan tim ―Indonesia juga sangat terpengaruh,
sukses adalah kelaziman. Sejak masa orde khususnya ekonomi. Persoalan ekonomi
lama hingga orde reformasi, bancakan Indonesia sebenarnya sudah bermasalah
jabatan dan kekuasaan sudah dan terus ditambah krisis yang terus mendera. Defisit
dilakukan. anggaran tahun 2020 melebar hingga 2,5%.
Namun di masa rezim Jokowi praktik Ini sungguh luar biasa memprihatinkan,‖
bancakan ini dilakukan begitu vulgar, tanpa ungkap Farid.
malu. Pertama, Jokowi sempat berjanji Farid pun menengarai bahwa problem
kabinetnya tidak akan diisi dengan praktik Corona tidak lepas dari kegagalan
bagi-bagi jabatan. Akhirnya ia membagi- kapitalisme global. Banyaknya jumlah
bagi jabatan seperti jabatan menteri dan korban dan kehilangan nyawa. Ini bukan
komisaris BUMN pada Partai Politik kali pertama. Sebelumnya kasus Ebola dan
pendukung dan tim suksesnya. Kedua, juga korban perang seperti di Suriah lebih
pemerintah Jokowi juga membagi jabatan 300 ribu kaum muslimin meninggal.
untuk puluhan perwira kepolisian. Kontras Karakter utama kapitalisme global yang
mendata ada tiga puluh perwira yang menjajah juga telah menewaskan 1 juta
mendapatkan jabatan di era Jokowi mulai orang di Irak. Dibandingkan dengan korban
dari komisaris hingga pejabat, di mana 21 Covid-19 masih sedikit dibandingkan
perwira masih aktif di kepolisian. dengan kepongahan pendudukan AS dan
Simpulan bacaan di atas adalah... negara penjajah di negeri kaum muslimin.
A. Di negara demokrasi, membagi-bagi Yang kedua, Corona telah
jabatan kepada parpol pendukung dan membongkar kegagalan ekonomi
tim sukses adalah kelaziman bahkan kapitalisme global ketika dihadapkan pada
dewasa ini semakin parah. goncangan wabah seperti ini. Pun demikian
B. Di negara demokrasi, membagi-bagi kapitalisme telah mengalami krisis dan
jabatan kepada parpol pendukung dan akan memperburuk perekonomian dunia.
tim sukses adalah kelaziman bahkan Negara-negara besar dunia pun sangat
begitu vulgar. lemah kebijakannya dalam masalah
C. Di Indonesia dan di semua negara ini,‖ungkap Farid yang juga sebagai
dengan sistem politik demokrasi saat ini pengamat politik nasional.
makin parah. Terkait isolasi atau lockdown, negara-
D. Di Indonesia membagi-bagi jabatan negara dunia perlu meniru Islam. Islam
kepada parpol pendukung dan tim mengisolasi wilayah yang menjadi sumber
sukses adalah kelaziman bahkan dewasa wabah tanpa mematikan produk ekonomi.
ini semakin parah. Sementara, di wilayah yang tidak menjadi
sumber wabah, kegiatan ekonomi

42
tetap produktif. Berbeda saat ini ketika Kalimat sumbang dalam paragraf tersebut
lockdown di seluruh wilayah perekonomian ditandai dengan nomor …
dan kegiatan seolah dimatikan dan dinon- A. (1)
aktifkan. Di akhir perbincangan Farid B. (2)
Wajdi menegaskan bahwa inti dari dampak C. (3)
Corona telah membuktikan kegagalan D. (4)
kapitalisme dunia dalam menjaga nyawa, E. (5)
kesehatan, dan kestabilan ekonomi.
15.Sekitar 1,3 miliar data pendaftaran kartu
Ide pokok tulisan di atas adalah.... SIM ponsel yang disinyalir bocor dan dijual
A. Corona mengkhawatirkan negara- di situs gelap dinilai Analis Media Sosial,
negara. Rizqi Awal, menjadi alarm bahaya bagi
B. Corona sangat mengkhawatirkan pengguna. ―Ini menjadi sesuatu alarm
negara. bahaya,‖ ujarnya kepada Mediaumat.id,
C. negara-negara khwatir Corona. Ahad (4/8/2022). ―Artinya, pemerintah
D. Corona sangat mengkhawatirkan bagi yang tadinya bilang bahwasanya tidak akan
banyak negara. ada kebocoran data, dan relatif akan aman,
E. negara sangat khwatir Corona. ternyata bisa dijebol loh data-data kita,‖
sambungnya.
13.Curhatan Jokowi terkait beratnya menahan Gagasan utama paragraf di atas adalah...
harga pertalite diangka Rp 7.650 per liter A. Sekitar 1,3 miliar data pendaftaran kartu
mengindikasikan adanya rencana rezim SIM ponsel disinyalir bocor dan dijual
untuk menaikkan harga BBM. Kenaikan itu di situs gelap.
akan menurunkan daya beli rakyat. Apalagi B. Sekitar 1,3 miliar data pendaftaran kartu
bagi 40 persen rakyat dengan pengeluaran SIM ponsel dinilai menjadi alarm
terbawah. “Hal ini akan berimbas pada bahaya bagi pengguna.
naiknya angka kemiskinan,” tegas Agung. C. Sekitar 1,3 miliar data pendaftaran kartu
Dampak buruk lainnya adalah UMKM SIM ponsel dinilai Analis Media Sosial.
berisiko tutup karena naiknya biaya D. Pemerintah menyampaikan tidak akan
produksi yang tak sanggup ditanggung. ada kebocoran data.
Menurutnya, hal ini akan berimbas pada E. Pemerintah menyampaikan data relatif
pemberhentian karyawan dan akan aman, ternyata bisa dijebol.
meningkatkan jumlah pengangguran.
Gagasan utama bacaan di atas adalah...
A. Jokowi hobi curhat
B. Curhatan Jokowi mengindikasikan.
C. Pemberhentian karyawan
D. Kenaikan BBM menurunkan daya beli
rakyat.
E. Meningkatkan jumlah pengangguran.

14.Cermatilah paragraf berikut dengan


saksama!
(1). Surat Keputusan Bersama (SKB) empat
menteri tentang upah minimum ditolak oleh
para pekerja di sektor swasta. (2)
Gelombang protes terjadi di mana-mana
bahkan tak jarang aksi itu diwarnai
kekerasan. (3) Pekerja menuntut besaran
gaji/upah yang tegas dari pemerintah. (4)
Hasil negosiasi antara wakil pekerja dengan
perusahaan dan dunia pengupahan tidak
disepakati (5) Hal ini dapat mengurangi
tingkat kemacetan yang ada di Jakarta.

43
A. (memandang)
B. Ah, sudahlah!
C. Kau juga mengalami seperti itu?
D. Terlalu sulit.
E. Sangat ruwet!

3. Alam : Jelas yang saya persalahkan


adalah sopir yang kurang waspada dalam
menjalankan tugasnya.
Asep : Ah, jangan sopir saja yang
dipersalahkan.
Lihat juga kondisinya.
Dodo : Benar, mungkin juga jalan di
situ licin.
Berdasarkan dialognya, tampak bahwa
ketiga tokoh itu mengungkapkan ...
A. egonya masing-masing
B. sikap yang mau menang sendiri
C. pendapat yamg berbeda mengenai suatu
persoalan
D. pertengkaran yang berawal dari adanya
IND-13 DRAMA perbedaan pendapat
KELAS XI E. perasaan yang terlalu dominan dalam
pendapat masing-masing
1. Narasi di awal cerita, kata-kata pembuka,
pengantar, ataupun latar belakang cerita 4. Rini : Pak Lukas memang guru sejati. Mau
yang biasanya disampaikan oleh narator melibatkan
atau dalang disebut … diri dengan problem anak-anaknya.
A. Dialog Dia seperti bapakku sendiri.
B. Resolusi Ani : Dia seorang Bapak yang melindungi,
C. Epilog sifatnya
D. Prolog lembut seperti seorang ibu.
E. Orientasi Tri : Bagaimana kalau dia kita juluki Pak
Lukas sang
2. Bacalah penggalan drama berikut ini! penyelamat.
Asdiarti : Kenapa? All : Setuju!
Yanti : Sangat ruwet! Penggalan dialog di atas menyatakan ...
Asdiarti : Kau dipaksa kawin? A. Pak Lukas guru rajin
Yanti : Antara lain itu. Banyak lagi. B. Pak Lukas guru sejati
Asdiarti : Apa? C. Pak Lukas guru yang disanjung
Yanti : Ah, sudahlah. Kau tak usah D. Pak Lukas guru yang dekat dengan
memaksaku murid
bicara. Sulit. Terlalu sulit. E. Pak Lukas guru yang menaruh perhatian
Asdiarti : Yah, aku tahu kau tak
kerasan di rumah. 5. Istri : Aku sudah tidak tahan!
Yanti : (memandang) Suami : Hai! (Memegang tangan
Asdiarti : Itu persoalan yang banyak istrinya).
kita rasakan Istri : Aku sangat bergembira, kita
bersama. akan mati
Yanti : Kau juga mengalami seperti bersama-sama…
itu? Suami : Tidak!
Istri : Jadi engkau tidak mau?
Bagian dialog yang mendukung watak Suami : Tidak, kita tidak boleh kalah
tokoh Yanti yang tertekan adalah ... dengan
kemiskinan

44
Tokoh istri dalam penggalan drama di atas B. kecewa
berwatak… C. marah
A. pesimis D. ambisius D. sedih
B. optimis E. materialis E. penasaran
C. antusias
9. Cecep : “Coba rokok tidak halal ini
Kutipan drama berikut digunakan untuk barangkali rasanya tidak enak daripada
soal nomor 6 dan 7. rokok halal kalau memangyang halal tidak
Abah : Kalau cari suami ada. Bukankah engkau sering mengatakan
harus yang jelas masa depannya, jangan cara kerjaku mendapat uang cara yang tidak
seperti si Kabayan! halal?
Iteung : Tapi Kang Kabayan mah Hakam : “Aku sudah merokok.”
baik nyah sama Iteung. Penuturan tokoh Cecep disampaikan
Abah : Baik? Baik apanya? dengan gaya…
Kalau memang baik…pasti suka ngirim A. Marah
uang, paling sedikit ngirim ikan B. putus asa
kesenangan Abah, ikan gurame! C. sombong
D. memaksa
6. Watak tokoh Abah adalah… E. menyindir
A. rendah hati
B. diktator 10. Bayu : “Sebenarnya ada apa?”
C. materialistis Edi : “Hari ini kan tanggal 10,
D. sabar uang SPP-ku
E. jujur belum dibayar. Bapakku
masih diluar kota, sedangkan
7. Tokoh Iteung pada kutipan drama di atas ibuku sakit sudah dua
akan lebih menarik jika menggunakan minggu. Aku malu pada
kostum… guruku”
A. Celana panjang, kaos dengan rambut (tertunduk)
panjang dibiarkan terurai Bayu : “Aku masih punya uang
B. Celana panjang dan kaos dengan rambut tabungan,
panjang dikepang dua. gunakanlah dulu! Jangan
C. Kebaya dan celana panjang dengan khawatir, kapan-kapan saja
rambut dibiarkan terurai. jika kau sudah punya uang
D. Kebaya dan kain dengan rambut baru dibayar.
dikepang dua. Dialog Edi pada cuplikan drama di atas
E. Kebaya dan kain dengan rambut pendek diucapkan dengan nada….
sebahu dibiarkan terurai. A. Sedih
B. riang
8. Ambu : Abah teh kesal sama Ambu? C. gembira
Cemburu? D. kaget
Abah : Cemburu? Abah cemburu E. memelas
sama Ambu?Tidak, tidak.
Ambu : Kalau tidak, kenapa atuh Penggalan drama digunakan untuk
tidak mau makan? menjawab soal nomor 1 s.d. 4!
Abah : Abah sudah tahu segalanya
tentang Ambu. Adegan Poirah dan Marni dengan
Ambu : Tahu apa? menggendong bakul dan mengenakan topi
Abah : Pokoknya rahasia Ambu aping.
sudah Abah pegang. Marni : Pon…Ponirah!
Ambu : Rahasia apa? Ponirah : Ada apa?
Abah : Ada aja! Marni : Aku melihat sepintas bayangan orang
Gaya dialog tokoh Ambu diucapkan di sana!
dengan nada… Ponirah : Tenang saja!
A. Kesal

45
Marni : Tenang…tenang! Tenang kurang tegas. Maaf, Pak Lurah kurang cek-
bagaimana? Kalau musuh? cek, kurang tepat.
Ponirah : Musuh? Marni, kita ini jualan buah Masalah yang diperdebatkan kedua tokoh
dan tidak punya musuh. Kita harus yakin, yang itu adalah, kecuali…
berani bergerak di malam hari hanya TNI. Ayo A. cara menyikapi suatu persoalan
jalan! B. peranan seorang lurah
Marni : Tapi bulu kudukku berdiri. C. ketepatan sikap Pak Lurah
Ponirah : Maka jangan di sini, ayo terus D. kekurangtegasan seorang lurah
jalan! E. perlunya pemikiran dalam menghadapi
Keduanya berjalan dengan sesekali menoleh suatu masalah
ke belakang. Topi aping di tangan kiri. Tangan
kanan di balik seakan memegang senjata.

11. Sikap Marnni yang tergambar dalam


adegan drama di atas adalah…
A. berani
B. ragu
C. terburu-buru
D. cemas
E. tenang

12.Penggalan adegan drama tersebut


menceritakan…
A. penyusupan ke tempat musuh
B. pelarian dua orang gadis dari
pengejaran musuh
C. pembelotan dua orang laksar dari
barak musuh
D. pengejaran tentara musuh terhadap dua
orang tentara
E. perjalanan malam dua orang tokoh
dalam suasana pertempuran

13.Watak Marni dalam adegan drama di atas


adalah…
A. Pemalu
B. pelupa
C. penakut
D. penyabar
E. pemarah

14.Ponirah menunjukkan sikap…


A. Tenang
B. acuh tak acuh
C. berani
D. tahu diri
E. pantang menyerah

15.L: Saya mesti tetap memikirkannya, Pak


Jagabaya. Sebagai seoranf lurah, saya tidak
akan berdiam diri menghdapi persoalan ini.
J: Tapi maaf, Pak Lurah, saya rasa tindakan
Pak Lurah dalam menghadapi persoalan ini

46
meningkat. (8) Meski berubah istilah,
pemerintah akan tetap menerapkan
pembatasan, misalnya pengunjung mal
dibatasi maksimum 75 persen dari kapasitas.
(9) Hanya warga yang sudah menjalani
vaksinasi dua dosis yang dapat beraktivitas di
tempat publik dan penerapan aplikasi Peduli-
Lindungi di ruang publik. (10) Ketua Bidang
Penanganan Kesehatan Satgas Covid-19
Nasional, Brigjen TNI Purn. dr. Alexander K.
Ginting, Sp.P(K)., FCCP mengatakan bahwa
ketentuan PPKM berpedoman pada Instruksi
Menteri Dalam Negeri Nomor 62 Tahun 2021
tentang Pencegahan dan
Penanggulangan Corona Virus Disease 2019
pada Saat Natal Tahun 2021 dan Tahun Baru
Tahun 2022. (11) Syarat perjalanan jarak jauh
dalam negeri saat Nataru adalah wajib
vaksinasi lengkap dan hasil antigen negatif
maksimal 1 × 24 jam sebelum keberangkatan.
(12) Untuk orang dewasa yang belum
mendapatkan vaksinasi lengkap ataupun yang
tidak bisa divaksin karena alasan medis dan
tak diizinkan untuk bepergian jarak jauh. (13)
Bagi anak-anak tetap dapat melakukan
perjalanan dengan syarat lebih ketat. (14)
IND-14 SEMANTIK Syaratnya telah melakukan PCR yang berlaku
KELAS XI 3 × 24 jam untuk perjalanan udara atau antigen
1 × 24 jam untuk perjalanan darat atau laut.
Bacalah teks berikut untuk menjawab soal 1. Kata bisa pada kalimat (12) berhomonim
nomor 1 dan 2! dengan kata bisa pada kalimat ….
(1) Menteri Dalam Negeri, Tito Karnavian, B. Setiap mahasiswa bisa mengajukan
mengatakan bahwa PPKM level 3 se- semester pendek di kampusnya.
Indonesia pada masa libur Nataru yang C. Supaya bisa melakukan aktivitas
dibatalkan pemerintah diganti dengan berikutnya, Anda harus menyelesaikan
pembatasan kegiatan masyarakat pada masa tugas sebelumnya.
Nataru. (2) Kebijakan itu diubah agar D. “Hore, sekarang saya bisa membuat
pembatasan yang diterapkan pada masa Nataru teleskop dari bahan sederhana!” kata
berlaku secara spesifik selama tanggal 24 Angga.
Desember 2021 hingga 2 Januari 2022, E. Sesampainya di gerbang objek wisata,
bergantung situasi di masing-masing daerah. Anda bisa langsung masuk dengan
(3) Tito menyampaikan ada beberapa faktor menunjukkan tiket.
yang membuat pemerintah membatalkan atau F. Kini bisa ular telah menjalar ke organ
menanggalkan rencana PPKM Level 3 se- tubuh lainnya.
Indonesia pada masa libur Nataru. (4) Pertama, 2. Berdasarkan kalimat nomor 2 dan 3
situasi pandemi Covid-19 di Indonesia dalam terdapat relasi makna jenis apa....
beberapa waktu terakhir relatif landai dan A. Polisemi
angka penularannya pun terbilang rendah. (5) B. Homonim
Berdasarkan survei yang dilakukan C. Homograf
Kementerian Kesehatan, antibodi masyarakat D. Antonim
sudah terbilang tinggi. (6) Bahkan, Tito E. Homofon
menyebut ada kemungkinan masyarakat di
sembilan daerah aglomerasi telah mengalami 3. Virus Corona yang menyambangi
kekebalan kelompok. (7) Kondisi daerah juga Indonesia sejak tahun lalu memberikan
selalu dinamis dan tingkat vaksinasi yang dampak besar. Salah satunya bagi pelaku

47
usaha UMKM yang harus gulung tikar. A. Mempersoalkan
Ketua Asosiasi UMKM Indonesia, B. lalu
(Akumindo), Ikhsan Ingratubun, C. sanksi
mengatakan selama tahun 2020 ada sekitar D. sebenar-benarnya
30 juta UMKM yang bangkrut karena E. bagi
Covid-19. Pada tahun 2019 jumlah UMKM
di Indonesia ada sebanyak 64,7 juta. 7. Kata yang memiliki hubungan homograf
Setelah terjadi pandemi Covid-19 jumlah terdapat pada kalimat...
UMKM di Indonesia menjadi 34 juta di A. Mereka paham bahwa dia hanya mereka
2020. saja soal -soal itu.
Frasa gulung tikar pada paragraf di atas B. Wanita yang menenteng keranjang itu
termasuk makna... sudah menuju ke ranjang.
A. Denotasi C. Guru sangsi atas bentuk sanksi yang
B. Leksikal ditimpakan kepada siswa yang bolos.
C. Konotasi D. Pawang itu bisa makan bisa ular sanca.
D. Sinonim E. Kakinya terperosok di kaki gunung itu.
E. Antonim
8. Kata-kata dalam kalimat berikut sama
4. Kata hitam yang bermakna denotasi bentuk dan ucapannya, tetapi berbeda
terdapat dalam kalimat.... maknanya, kecuali....
A. Ketika melihat temannya yang berkulit A. Setelah pemburu itu pasang perangkap,
hitam itu, jantungnya berdegup keras. ia mendapat dua pasang bajing.
B. Jangan sampai permasalahn ekonomi B. Dia tidak bisa mendapatkan penawar
membuat kita terjerumus ke lembah bisa ular itu.
hitam. C. Pada tanggal dua, gigi adik yang kedua
C. Ilmu hitam yang dipakainya sangat juga tanggal.
membahayakan dirinya. D. Di tambang itu banyak terdapat tambang
D. Ketika orang yang dibencinya lewat, yang masih baru.
hitamlah mukanya. E. Pak Bejo dapat menjual 50 pack rokok
E. Hitam putih kehidupanmu sangat setiap hari.
tergantung pada pemahaman kita
tentang agama. 9. Bacalah teks berikut dengan cermat!
Semangatnya telah padam.
5. Air terjun banyak dijadikan sumber energi. Api di tungku telah padam.
(2) Aliran air yang sangat deras dapat Kemauannya telah padam.
menggerakkan kincir. (3) Kincir air akan Kata “padam” pada ketiga kalimat di atas
memutar dinamo sebagai penghasil energi berelasi secara....
listrik. (4) Energi listrik yang dihasilkan A. Homonim
dari aliran air deras sangat menguntungkan. B. Sinonim
(5) Pasang surutnya air sangat menentukan C. Homofon
adanya listrik tenaga air. D. Polisemi
Kalimat yang menggunakan kata E. Homograf
berantonim di dalam paragraf tersebut
ditandai dengan nomor .... 10. Bacalah penggalan teks beriku!
A. (1) Baru-baru ini beredar berita mengenai
B. (5) Risma memberikan pekerjaan pada 15
C. (3) orang tunawisma di perusahaan BUMN PT.
D. (4) Waskita Karya. Risma pun mengantarkan
E. (2) para tunawisma ke tempat kerjanya, yakni
di lokasi proyek Tol Becakayu, Bekasi,
6. Kita tidak mempersoalkan beratnya sanksi Jawa Barat (21/01). Risma juga dikabarkan
bagi pelanggar peraturan lalu lintas karena memberikan pekerjaan kepada lima
hal itu sebenar- benarnya. tunawisma di anak perusahaan BUMN PT
Pada kalimat di atas, manakah kata yang PP Properti TBK di Grand Kamala Lagoon,
termasuk homofon? Bekasi. Karena kebijakan dan keputusan

48
Risma tersebut, Ketua DPP Partai Keadilan Karantina Tumbuhan, yang bertempat di
Sejahtera (PKS) Bukhori Yusuf menilai lapangan kantor Karantina Pertanian
langkah Menteri Sosial Tri Rismaharini Medan. Dan kegiatan dilakukan dengan
memberikan pekerjaan kepada sejumlah mematuhi Protokol Kesehatan.
tunawisma di perusahaan BUMN berkaitan Kata “apel” pada kalimat di atas jika
dengan Pilkada DKI Jakarta mendatang. dipadankan dengan kata “apel” pada
Kata “tunawisma” pada teks tersebut kaliamt Saya membeli jeruk, anggur, dan
mengalami perubahan makna…. apel di pasar buah kemarin. Maka relasi
A. Ameliorasi makna yang terjadi adalah….
B. Antonim A. Polisemi
C. Peyorasi B. Homonim
D. Sinestesia C. Homograf
E. Sinonim D. Homofon
11. Kata “rumah” dalam kalimat berikut telah E. Hiponim
mengalami proses tata bahasa (makna 14. Makna kata bergaris bawah berikut
gramatikal), kecuali.... mengalami perubahan peyoratif….
A. Riana sedang membangun rumah A. Laki-laki itu akan mengantar istrinya.
impiannya. B. Ibu muda itu sedang hamil tua.
B. Pejabat itu dirumahkan setelah terbukti C. Ani memberikan pakaian yang bagus
bersalah. kepada tunawisma itu.
C. Seorang ibu merupakan rumah D. Dia sedang kesal, karena lakinya
pertama bagi anaknya. belum pulang.
D. Terlihat rumah-rumah adat E. Dia memang suami yang baik.
mengelilingi kawasan itu. 15. Perhatikan wacana berikut!
E. Dia dilarikan ke rumah sakit terdekat. Belum pulih dari banjir bandang
12. Kelompok kata berikut mengalami makna beberapa waktu lalu, Kecamatan
ameliorasi adalah.... Masamba, Kabupaten Luwu Utara,
A. Tunarungu, tunasusila, miskin, Sulawesi Selatan kembali dilanda banjir,
penjara. Selasa (4/8/2020) malam. Banjir susulan
B. Tunakarya, tunawisma, prasejahtera, terjadi karena hujan deras mengguyur
bisu. bagian hulu Kecamatan Masamba sejak
C. Buta huruf, tuli, gelandangan, Selasa siang, yang membuat Sungai
kesalahan prosedur. Masamba meluap dan merendam
D. Tunawicara, bini, beranak, pembantu. permukiman warga di Kelurahan Bone
E. Tutup usia, diamankan, dirumahkan, Tua, Kelurahan Bone dan Desa Baloli
penyesuaian harga. dengan ketinggian banjir bervariasi antara
13. Mengawali kegiatan, rutinitas apel pagi 50 hingga 80 sentimeter.
kembali dilaksanakan oleh seluruh Kepala Pelaksana BPBD Luwu Utara,
pegawai Karantina Pertanian yang Muslim Muchtar, mengatakan selain
dipimpin langsung oleh Kepala Kantor pemukiman warga, banjir juga merendam
drh, Putu Gede Widiarsa Putra,M.Si jalan Trans Sulawesi Masamba, yang
(01/02). Pada kesempatan ini Putu Gede membuat kendaraan tak dapat melintas.
menghimbau kepada seluruh pegawai Muslim mengatakan, di sejumlah titik
Karantina Pertanian Medan maupun permukiman warga terendam hingga satu
tenaga kontrak (honor) untuk wajib meter, yang membuat warga mengungsi
melakukan rutinitas apel pagi yaitu ke tempat yang lebih aman. Banjir yang
sebagai wujud cerminan disiplin pegawai. terjadi saat ini merupakan banjir susulan
Putu Gede juga menyampaikan agar setelah sebelumnya pada tiga pekan lalau
seluruh pegawai bekerja dengan lebih diterjang banjir setinggi dua meter yang
maksimal khususnya dalam melayani membawa material berupa pasir
masyarakat. Mari kita berupaya bercampur kayu. Sejumlah relawan yang
meningkatkan kedisiplinan dan etos masih berada di lokasi pengungsian
kerjanya, sambungnya. Kegiatan apel pagi seperti PMI, ACT dan lainnya kini
dihadiri pejabat struktural dan Sub melakukan evakuasi terhadap warga
Koordinator Karantina Hewan dan bersama BPBD Luwu Utara.

49
Kata kepala pada kalimat pertama D. S-P-O-K
paragraf kedua dalam wacana di atas E. S-P-K
dengan kalimat Rudi terjatuh hingga
kepalanya terluka mengandung relasi 2. Berdasarkan kalimat keenam, pola yang
makna…. merupakan frasa, kecuali....
A. Homonim A. Predikat
B. Homograf B. Keterangan tujuan
C. Homofon C. Subjek
D. Hipernim D. Objek
E. Polisemi E. Keterangan tempat

3. Berdasarkan kalimat kelima, berapakah


jumlah frasa yang terdapat pada kalimat
tersebut?
A. 4 frasa
B. 3 frasa
C. 2 frasa
D. 1 frasa
E. 0 frasa

4. Berdasarkan kalimat kelima, pola predikat


diduduki oleh kata?
A. berbentuk ruangan
B. ruang baca
C. ruang refernsi
D. terbagi
E. perpustakaan

IND-15 SINTAKSIS Perhatikan teks berikut untuk


KELAS XI menjawab soal nomor 5 s.d 8!
(1) Sampah merupakan material sisa yang
Bacalah paragraf di bawah ini dengan tidak diinginkan setelah berakhirnya suatu
saksama untuk menjawab soal nomor 1 s.d. proses. (2) Sampah dapat bersumber dari
4! alam, manusia, konsumsi, nuklir, industri,
(1) Perpustakaan adalah tempat yang dan pertambangan. (3) Sampah di bumi
disediakan untuk pemeliharaan dan akan terus bertambah jika masih ada
penggunaan koleksi buku. (2) Perpustakaan kegiatan yang dilakukan. (4) Sampah yang
berbentuk ruangan dan gedung. (3) dihasilkan di Indonesia mencapai jumlah
Perpustakaan yang berbentuk ruangan 11.330 ton per hari. (5) Sampah dapat
biasanya terdapat di sekolah-sekolah. (4) dibedakan berdasarkan sifat dan bentuknya.
Sedangkan yang berbentuk gedung biasnya (6) Berdasarkan sifatnya, sampah bibagi
terletak di kota (Perpustakaan Daerah). (5) menjadi dua, yaitu sampah organik dan
Perpustakaan yang berbentuk ruangan terbagi anorganik.
atas ruang baca dan ruang referensi. (6) Di
bagian dalam, ruang baca memiliki banyak 5. Kalimat yang sepola dengan kalimat kedua
meja dan kursi untuk membaca. (7) Sedangkan paragraf di atas adalah...
ruang referensi adalah tempat berbagai rak A. Kami berlindung di bawah pohon
buku yang dikelompokkan menurut jenis-jenis rindang.
buku tersebut. B. Rumah kami terbakar kemarin.
C. Jika hari hujan, saya tidak datang.
1. Berdasarkan kalimat ketiga, apakah pola D. Yang duduk di sana adalah teman
yang terdapat pada kalimat tersebut? baikku.
A. S-P-O-K-K E. Laut itu sangat dalam.
B. P-S-O-K
C. K-S-P-O-P

50
6. Pola yang terdapat pada kalimat ketiga (1) Lautan Sabang adalah pintu masuk ke
adalah.... Selat Malaka. (2) Di lepas laut, sesekali
A. K-P-S-O kapal tanker atau kapal bermuatan
B. S-K-P-K kontainer lewat. (3) Dulu saat Perang
C. S-P-O Dunia 2, kapal yang lewat adalah kapal-
D. S-P-K kapal perang Amerika dan sekutunya. (4)
E. P-S-O-K Meriam jenis coastal defense
milik Jepang tersebut menembakan peluru
7. Berdasarkan kalimat ketiga, pola subjek ke arah kapal-kapal itu agar mereka tak
diduduk oleh kata.... memasuki Selat Malaka.
A. sampah 11. Kalimat yang sepola dengan kalimat
B. terus bertambah bernomor 4 adalah....
C. kegiatan yang dilakukan A. Kesehatan merupakan pilar utama agar
D. sampah di bumi manusia semangat menjalani hari.
E. jika masih ada B. Dampak perundungan dapat dilihat
langsung melalui sikap koran yang
8. Berdasarkan kalimat ketiga, berapa jumlah dirundung.
frasa yang terdapat dalam kalimat C. Pekan depan, pemerintah berencana
tersebut .... menaikkan harga BBM.
A. 5 frasa D. Keputusan yang dikeluarkan pemerintah
B. 4 frasa tersebut dinilai tidak baik oleh pengamat
C. 3 frasa ekonomi.
D. 2 frasa E. Kenyamanan kelas di sekolah menjadi
E. 1 frasa motivasi bagi seluruh siswa untuk tetap
fokus dalam belajar.
9. Sumber daya alam Indonesia dikuasai
orang asing sejak dahulu kala. Semua pola 12.Berdasarkan paragraf di atas, kalimat yang
kalimat tersebut adalah frasa, kecuali.... memiliki pola inversi terdapat pada kalimat
A. predikat nomor....
B. objek A. (1)
C. subjek B. (2)
D. keterangan C. (3)
E. subjek dan keterangan D. (4)
E. (1) dan (4)
10. (1) Lautan Sabang adalah pintu masuk ke
Selat Malaka. (2) Di lepas laut, sesekali 13.Berdasarkan paragraf di atas, kalimat yang
kapal tanker atau kapal bermuatan memiliki pola S-P-K adalah kalimat
kontainer lewat. (3) Dulu saat Perang nomor....
Dunia 2, kapal yang lewat adalah kapal- A. (1)
kapal perang Amerika dan sekutunya. (4) B. (2)
Meriam jenis coastal defense C. (3)
milik Jepang tersebut menembakan peluru D. (4)
ke arah kapal-kapal itu agar mereka tak E. (1) dan (4)
memasuki Selat Malaka.
Pernyataan di bawah ini BENAR Perhatikan teks berikut untuk
mengenai kalimat bernomor 3, kecuali.... menjawab soal nomor 4 dan 5!
A. Memiliki pola K-S-P Bulan adalah satelit alami yang dimiliki
B. Merupakan kalimat normal Bumi yang menjadi nomor 5 satelit terbesar
C. Memiliki tiga frasa di tata surya. Bulan melakukan rotasi atau
D. S dalam kalimat tersebut mengalami putaran untuk mengelilingi bumi. Satu kali
perluasan putaran kita kenal dengan 1 bulan. Kita
E. Merupakan kalimat inversi hanya dapat melihat bulan ketika malam
hari. Ketika malam hari, bulan nampak
Bacalah paragraf berikut ini untuk terlihat mengeluarkan cahaya, namun
menjawab soal nomor 1 s.d. 3! sebenarnya cahaya yang dikeluarkan oleh

51
bulan bukanlah cahayanya sendiri,
melainkan berasal dari cahaya matahari.

14.Berdasarkan kalimat kedua penggalan teks


di atas, manakan pernyataan berikut yang
TIDAK BENAR...
A. Memiliki pola S-P-O-K
B. Berjenis kalimat normal
C. Objek kalimat tersebut adalah rotasi
atau putaran
D. Keterangan kalimat tersebut adalah
rotasi atau putaran mengelilingi bumi
E. Kata bulan berjabatan subjek.

15.Berdasarkan kalimat kesatu, pola kalimat


tersebut adalah....
A. S-P-K-K
B. P-S-O-K
C. S-P-O-K
D. S-P-O
E. S-P-K

52

Anda mungkin juga menyukai